Đến nội dung

Hình ảnh

Tổng hợp các bài BĐT

ẩn đi không cho xem

  • Please log in to reply
Chủ đề này có 164 trả lời

#1
CD13

CD13

    Thượng úy

  • Thành viên
  • 1456 Bài viết

Topic này chủ yếu là coppy nội dung đề bài và lời giải của các mem lại, các ĐHV THPT có thể coppy tiếp CD13 nhưng nên để chung trong một khung. Tính đúng sai của các lời giải thì kiểm tra lại sau.

 

Bắt đầu vậy!

 

Bài 1:

Cho abc=1 va $a^{3}> 36.  CMR  :\frac{a^{2}}{3}+b^{2}+c^{2}> ab +bc+ca$}

Lời giải:

$VT-VP=\frac{a^{2}}{4}+b^{2}+c^{2}-ab-bc+2bc+\frac{a^{2}}{12}=(\frac{a}{2}-b-c)^{2}+\frac{a^{2}-36bc}{12}>0\Rightarrow$ đpcm

Cách khác:

Từ giả thiết suy ra $a>0$ và $bc>0$. Bất đẳng thức cần chứng minh tương đương với
\[\dfrac{a^2}{3}+(b+c)^2-3bc-a(b+c)\ge 0\\ \iff \dfrac{1}{3}+\left(\dfrac{b+c}{a}\right)^2-\dfrac{b+c}{a}-\dfrac{3}{a^3}\ge 0\]
Vì $a^3>36$ nên \[\dfrac{1}{3}+\left(\dfrac{b+c}{a}\right)^2-\dfrac{b+c}{a}-\dfrac{3}{a^3}> \left(\dfrac{b+c}{a}\right)^2-\dfrac{b+c}{a}+ \dfrac{1}{4}= \left(\dfrac{b+c}{a}-\dfrac{1}{2}\right)^2 >0\]
 
 

 

Bài 2:

Với a,b,c >0; n ∈ N*.CMR:

$\frac{a^{n}}{b+c}+\frac{b^{n}}{a+c}+\frac{c^{n}}{a+b}\geq \frac{3}{2}\left ( \frac{a^{n}+b^{n}+c^{n}}{a+b+c} \right )$

Lời giải:

$\sum \frac{a^{n}}{b+c}\geq \frac{1}{3}(\sum a^{n})(\sum \frac{1}{a+b})\geq \frac{1}{3}(\sum a^{n})(\frac{9}{2(a+b+c)})=\frac{3}{2}(\frac{\sum a^{n}}{\sum a})$

 

 

 

Bài 3:

Cho $x,y,z >0$ thỏa điều kiện $x^{2}+y^{2}+z^{2}=9$

Tìm giá trị nhỏ nhất của $P=\frac{x^5}{y^2}+\frac{y^5}{z^2}+\frac{z^5}{x^2}$

Lời giải:

Theo $Cauchy$ Ta có:

$$\dfrac{x^5}{y^2}+\dfrac{x^5}{y^2}+\sqrt{3}y^2+\sqrt{3}y^2+3\sqrt{3}\ge \sqrt{3}x^2$$

Cách khác:

Sử dụng Cauchy-Schwarzt ta có 

       $\frac{x^5}{y^2}+\frac{y^5}{z^2}+\frac{z^5}{x^2}\geqslant \frac{(x^3+y^3+z^3)^2}{xy^2+yz^2+zx^2}$

Sử dụng Cauchy-Schwarzt và AM-GM ta có 

       $xy^2+yz^2+zx^2\leqslant \sqrt{(x^2+y^2+z^2)(x^2y^2+y^2z^2+z^2x^2)}\leqslant \sqrt{\frac{(x^2+y^2+z^2)^3}{3}}=3$

Do đó $P\geqslant \frac{(x^3+y^3+z^3)^2}{3}\geqslant \frac{(x^2+y^2+z^2)^3}{9}=3$ 

Đẳng thức xảy ra khi $x=y=z=1$

 

 

 

Bài 4:

Ch0 $a>0$ và $n$ là 1 số tự nhiên

Chứng minh rằng $a^n+\frac{1}{a^n}-2\geqslant n^2(a+\frac{1}{a}-2)$

Lời giải:

Bất đẳng thức tương đương với $(a^{n-1}+a^{n-2}+...+a+1)\geq n^2a^{n-1}$ (hiển nhiên theo AM-GM)

Cách khác:

Do tính đối xứng giữa a và $\frac{1}{a}$ nên ta có thể giả sử a ≥ 1.  đặt $\sqrt{a}$ =x ≥ 1.bdt $\Leftrightarrow$ $x^{2n}+\frac{1}{x^{2n}}-2 \geq n^{2}(x^{2}+\frac{1}{x^{2}}-2)\Leftrightarrow (x^{n}-\frac{1}{x^{n}})^{2}\geq n^{2}(x-\frac{1}{x})^{2} \Leftrightarrow $x^{n}-\frac{1}{x^{n}}\geq n(x-\frac{1}{x})$①.

Với x=1 thì ① đúng

Với x>1 thì ① $\Leftrightarrow x^{n-1} +x^{n-3} ...+\frac{1}{x^{n-3}}+\frac{1}{x^{n-1}}\geq n$ (đúng vì theo bđt AM-GM).

Dấu bằng xảy ra khi x=1 $\Leftrightarrow a=1$

 

 

 

Bài 5:

Cho $a,b,c,d$ là các số thực thỏa mãn $\left\{\begin{matrix} a+b+c+d=0\\a^2+b^2+c^2+d^2=2 \end{matrix}\right.$

Tìm GTLN của $P=abcd$

Lời giải:

Áp dụng AM-GM ta có 

$2=\sum a^{2}\geq 4\sqrt[4]{\prod a^{2}}\Rightarrow \sqrt{\left | abcd \right |}\leq \frac{1}{2}\Rightarrow abcd\leq \frac{1}{4}$

Dấu bằng xảy ra khi $a=b=-c=-d=\frac{1}{\sqrt{2}}$ và các hoán vị của chúng

 

 

 

Bài 6:

Cho $a,\,b,\,c\geq 0$ thỏa mãn $a+b+c=1.$ Tìm giá trị lớn nhất của biểu thức: $$P=abc\left(a^2+b^2+c^2\right)$$

Lời giải:

Ta có: $P=abc(a+b+c)(a^{2}+b^{2}+c^{2})\leq \frac{1}{3}(ab+bc+ca)^{2}(a^{2}+b^{2}+c^{2})$

 

Mặt khác, lại có: $(ab+bc+ca)^{2}(a^{2}+b^{2}+c^{2})\leq \left (\frac{(a+b+c)^{2}}{3} \right )^{3}=\frac{1}{27}$

 

Do đó: $P\leq \frac{1}{81}$

 

Dấu bằng xảy ra khi và chỉ khi $a=b=c= \frac{1}{3}$

 

 

 

Bài 7:

Cho các số thực $x,\,y>0$ thỏa mãn $3x+y\leq1.$ Tìm giá trị nhỏ nhất của biểu thức: $$S=\dfrac{1}{x}+\dfrac{1}{\sqrt{xy}}$$

Lời giải:

$S\geq \frac{1}{x}+\frac{1}{\sqrt{x(1-3x)}}$

$\geq \frac{1}{x}+\frac{2}{1-2x}=\frac{2}{x(1-x)}\geq \frac{8}{(x+1-x)^{2}}=8$

Dấu "=" xảy ra $\Leftrightarrow x=\frac{1}{4}$

 

 

 

Bài 8:

Cho các số thực a,b,c,x,y thỏa mãn $$ax-by=\sqrt{3}$$ .

Tìm GTNN của $F= a^{2}+b^{2}+x^{2}+y^{2}+ bx +ay$

Lời giải:

Sử dụng giả thiết $ax-by=\sqrt{3}$ ta có:
$$(a^2+b^2)(x^2+y^2)=(ax+by)^2+(ax-by)^2=(ax+by)^2+3$$
Áp dụng bất đẳng thức $Cauchy$ , suy ra:
$$a^2+b^2=x^2+y^2=(a^2+b^2)+(x^2+y^2) \\ \ge 2\sqrt{(a^2+b^2)(x^2+y^2)}=2\sqrt{(ax+by)^2+3}$$
Do đó, ta đưa về bài toán tìm GTNN của: $2\sqrt{x^2+3}+x$ trong đó $x=ax+by$
Ta có:
$$\left(2\sqrt{x^2+3}+x\right)^2=4(x^2+3)+4x\sqrt{x^2+3}+x^2 \\ = (x^2+3)+4x\sqrt{x^2+3}+4x^2+9 \\ = \left(\sqrt{x^2+3}+2x\right)^2+9\ge 9$$
$$\Rightarrow 2\sqrt{x^2+3}+x\ge 3$$
Vậy $\text{MinT}=\fbox{3}$
 
 
 

Bài 9:

Cho các số thực dương $a,b,c$. Tìm giá trị nhỏ nhất của biểu thức:

 

                                $P=\frac{2}{a+ \sqrt{ab}+ \sqrt[3]{abc}}-\frac{3}{\sqrt{a+b+c}}$

Lời giải:

$a+\sqrt{\frac{1}{2}a.2b}+\sqrt[3]{\frac{1}{4}a.b.4c}\leq a+\frac{1}{4}a+b+\frac{1}{12}a+\frac{1}{3}b+\frac{4}{3}c=\frac{4}{3}(a+b+c)$

Do đó $P\geq \frac{3}{2(a+b+c)}-\frac{3}{\sqrt{a+b+c}}$...

 

 

 

Bài 10:

Cho x,y là các số không âm thoả $x^{3}+y^{3}\leq 1$

 

Tìm giá trị lớn nhất của $P=2\sqrt{x}+\sqrt{y}$

Lời giải:

$(x^3+y^3)(\sqrt[5]{2^6}+1)^5\geqslant (2\sqrt{x}+\sqrt{y})^6$

$\Leftrightarrow 2\sqrt{x}+\sqrt{y}\leqslant \sqrt[6]{(\sqrt[5]{2^6}+1)^5}$

Vậy $Max(P)= \sqrt[6]{(\sqrt[5]{2^6}+1)^5}\Leftrightarrow \frac{a^3}{2\sqrt[5]{2}}=b^3=\frac{1}{2\sqrt[5]{2}+1}$

 

Chiều coppy tiếp, sau đó kiểm tra nội dung sau.

 

 

Bài 11:Cho các số a,b,c không âm không đồng thời bằng không. Chứng minh rằng;

$\sum \frac{2a^{2}-bc}{b^{2}-bc+c^{2}}\geq 3$

 

Lời giải:(vutuanhien)

 

Không mất tính tổng quát, ta có thể giả sử $b$ là số nằm giữa $a$ và $c$

BĐT đã cho tương đương với

$$\sum \frac{2a^2+(b-c)^2}{b^2-bc+c^2}\geq 6$$

Áp dụng BĐT Cauchy-Schwarz, ta có

$$\sum \frac{2a^2}{b^2-bc+c^2}\geq \frac{2(a^2+b^2+c^2)^2}{\sum a^2(b^2-bc+c^2)}=\frac{2(a^2+b^2+c^2)^2}{2\sum a^2b^2-abc\sum a}$$

$$\sum \frac{(b-c)^2}{b^2-bc+c^2}\geq \frac{[a(b-c)+b(a-c)+c(a-b)]^2}{2\sum a^2b^2-abc\sum a}=\frac{4b^2(a-c)^2}{2\sum a^2b^2-abc\sum a}$$

Do đó ta chỉ cần chứng minh

$$(a^2+b^2+c^2)^2+2b^2(a-c)^2\geq 6\sum a^2b^2-3abc\sum a (1)$$

Ta có 

$b^2(a-c)^2=[a(b-c)+c(a-b)]^2=a^2(b-c)^2+c^2(a-b)^2+2ac(a-b)(b-c)$

$\geq a^2(b-c)^2+c^2(a-b)^2$

Suy ra 

$$2b^2(a-c)^2\geq a^2(b-c)^2+b^2(c-a)^2+c^2(a-b)^2$$

$$\Rightarrow VT (1)\geq (\sum a^2)^2+2\sum a^2b^2-2abc\sum a$$

Do đó ta chỉ còn phải chứng minh 

$$(\sum a^2)^2+2\sum a^2b^2-2abc\sum a\geq 6\sum a^2b^2-3abc\sum a$$

$$\Leftrightarrow \sum a^4+abc\sum a\geq 2\sum a^2b^2$$

BĐT này hiển nhiên đúng theo BĐT Schur

$$\sum a^4+abc\sum a\geq \sum ab(a^2+b^2)$$

Và BĐT AM-GM

$$\sum ab(a^2+b^2)\geq 2\sum a^2b^2$$

Kết thúc chứng minh 

Đẳng thức xảy ra khi $a=b=c$ hoặc $a=b$, $c=0$ và các hoán vị.

 

Bài 12:(bosulan239)

Cho a,b,c là các số không âm không đồng thời bằng không.

CMR

$\frac{\sum a^{2}}{\sum ab}\geq \sum \frac{ab}{b^{2}+bc+c^{2}}$

Bài giải:(vutuanhien)

 

BĐT đã cho tương đương với

$\frac{a^2}{ab+bc+ca}-\frac{ab}{b^2+bc+c^2}+\frac{b^2}{ab+bc+ca}-\frac{bc}{c^2+ca+a^2}+\frac{c^2}{ab+bc+ca}-\frac{ca}{a^2+ab+b^2}\geq 0$

$\Leftrightarrow \sum \frac{ac(ac-b^2)}{b^2+bc+c^2}\geq 0$

Do $\frac{ac(ac-b^2)}{b^2+bc+c^2}=\frac{ac^2(a+b+c)}{b^2+bc+c^2}-ac$ nên BĐT đã cho có thể viết lại thành

$\sum \frac{ac^2(a+b+c)}{b^2+bc+c^2}\geq ab+bc+ca$

$\Leftrightarrow \sum \frac{ac^2}{b^2+bc+c^2}\geq \frac{ab+bc+ca}{a+b+c}$

Áp dụng BĐT Cauchy-Schwarz, ta có

$VT\geq \frac{(ab+bc+ca)^2}{\sum a(b^2+bc+c^2)}=\frac{ab+bc+ca}{a+b+c}$

Kết thúc chứng minh 

 

 


Bài 13: (nguyencuong123)

Cho a,b,c không âm thoả mãn: $a+b+c=3$

Chứng Minh: $\sum \frac{a+1}{ab+1}\geq 3$

 

 

Bài giải:(Juliel)

Áp dụng AM-GM cho vế trái, ta cần chứng minh :

$(a+1)(b+1)(c+1)\geq (ab+1)(bc+1)(ca+1)\Leftrightarrow abc+(ab+bc+ca)+(a+b+c)+1\geq a^{2}b^{2}c^{2}+abc(a+b+c)+(ab+bc+ca)+1\Leftrightarrow abc+4\geq a^{2}b^{2}c^{2}+3abc+1\Leftrightarrow a^{2}b^{2}c^{2}+2abc\leq 3$

Hiển nhiên đúng vì $abc\leq (\frac{a+b+c}{3})^{3}=1$

 


 


Bài 14:(Chrome98):Chứng minh bất đẳng thức sau với $a,b,c>0$ và $a+b+c=1$:

 

\[ \frac{a^2}{3a+1}+\frac{b^2}{3b+1}+\frac{c^2}{3c+1}\ge 24\left(\frac{a^2}{9a+1}+\frac{b^2}{9b+1}+\frac{c^2}{9c+1}\right)^2 \]

Bài giải: (Simpson Joe Donald)

 

 
 
$\bullet\ AM-GM:\ \dfrac{a^2}{9a+1}= \dfrac{a^2}{6a+(3a+1)}\le \dfrac{a^2}{2\sqrt{6a(3a+1)}}= \dfrac{a\sqrt{a}}{2\sqrt{6(3a+1)}}$ ;
$\bullet\ Cauchy-Schwarz:\ VP\le \left( \dfrac{a\sqrt{a}}{\sqrt{3a+1}}+  \dfrac{b\sqrt{b}}{\sqrt{3b+1}}+  \dfrac{c\sqrt{c}}{\sqrt{3c+1}}\right)^2\le (a+b+c).VT=VT$

 

Bài 15:(trauvang97:)Cho các số thực dương $a,b,c$ thoả mãn:

 

                             $\frac{2}{a^{2}+1}+\frac{2}{b^{2}+1}+\frac{2}{c^{2}+1}\geq 3$

 

Chứng minh rằng: $(a-2)^{2}+(b-2)^{2}+(c-2)^{2}\geq 3$

 

Bài giải:

 

(Nguyen Huy Tuyen)$\frac{2}{a^{2}+1}+\frac{2}{b^{2}+1}+\frac{2}{c^{2}+1}\geq 3\Leftrightarrow \sum \frac{(1-a)(1+a)}{a^2+1}\geqslant 0$

$(a-2)^{2}+(b-2)^{2}+(c-2)^{2}-3=\sum (a-3)(a-1)$

Ta có :$\sum (a-3)(a-1)-\sum \frac{2(1-a)(1+a)}{a^2+1}=\sum \frac{(a-1)^4}{a^2+1}\geqslant 0$

           $\Leftrightarrow \sum (a-3)(a-1)\geqslant \sum \frac{2(1-a)(1+a)}{a^2+1}\geqslant 0$

           $\Leftrightarrow (a-2)^{2}+(b-2)^{2}+(c-2)^{2}\geq 3$

 

 

Bài 16:(phanquockhanh)Cho $x,y,z >0 : xyz+x+z=y$ . Tìm giá trị lớn nhất của biểu thức : $P=\frac{2}{x^2+1} - \frac{2}{y^2+1} -\frac{4z}{\sqrt{z^2+1}}+\frac{3z}{(z^2+1).\sqrt{z^2+1}}$

(Trích đề thi thử số 2 – THTT)

Bài giải:

(trauvang97)Từ giả thiết ta có: $x=\frac{y-z}{1+yz}$.

 

Khi đó:

 

$P=\frac{2(1+yz)^{2}}{(y^{2}+1)(z^{2}+1)}-\frac{2}{y^{2}+1}-\frac{4z}{\sqrt{z^{2}+1}}+\frac{3z}{(z^{2}+1)\sqrt{z^{2}+1}}$

 

$P=\frac{2z(2y+(y^{2}-1)z)}{(y^{2}+1)(z^{2}+1)}-\frac{4z}{\sqrt{z^{2}+1}}+\frac{3z}{(z^{2}+1)\sqrt{z^{2}+1}}$

 

Do $\frac{2z(2y+(y^{2}-1)z)}{(y^{2}+1)(z^{2}+1)}=\frac{2z\sqrt{(2y+(y^{2}-1)z)^{2}}}{(y^{2}+1)(z^{2}+1)}\leq \frac{2z\sqrt{(4y^{2}+(y^{2}-1)^{2})(1+z^{2})}}{(y^{2}+1)(z^{2}+1)}=\frac{2z}{\sqrt{z^{2}+1}}$

 

Do đó:

 

$P\leq \frac{2z}{\sqrt{z^{2}+1}}-\frac{4z}{\sqrt{z^{2}+1}}+\frac{3z}{\sqrt{z^{2}+1}}\left ( 1-\frac{z^{2}}{z^{2}+1} \right )$

 

$P=-3t^{3}+t$ với $\frac{z}{\sqrt{z^{2}+1}}=t\in (0;1)$

 

Khảo sát hàm số trên ta thấy $maxP=\frac{2}{9}\Leftrightarrow x=\frac{\sqrt{2}}{2};y=\sqrt{2},z=\frac{\sqrt{2}}{4}$

 

 

 


Bài 17:(Toc Ngan)Cho $a,b,c >0$ và $a+b+c=3$

Chứng minh rằng : $8(\frac{1}{a}+\frac{1}{b}+\frac{1}{c})+9 \geq 10(a^2+b^2+c^2)$

 Bài giải:(babystudymaths)

 

Cách 1

Giải như sau:

Giả sử a là số lớn nhất trong 3 số a,b,c ,thế thì c nhỏ hơn 3 và không nhỏ hơn 1

ta thấy $9=(42a-48)+(42b-\frac{69}{2})+(42c-\frac{69}{2})$

Thay và BĐT ban đầu ta thấy tương đương

$(\frac{8}{b}-10b^{2}+42b-\frac{69}{2})+(\frac{8}{c}-10c^{2}+42c-\frac{69}{2})\geq 10a^{2}-\frac{8}{a}-42a+48\Leftrightarrow \frac{(16-5b)(2b-1)^{2}}{b}+\frac{(16-52)(2c-1)^{2}}{c}\geq \frac{4(5a-1)(a-2)^{2}}{a}$

Áp dụng BCS ,ta có:

VT $\geq \frac{(2b-1+2c-1)^{2}}{\frac{b}{16-5b}+\frac{c}{16-5c}}= \frac{4(a-2)^{2}}{\frac{b}{16-5b}+\frac{c}{16-5c}}$

Lúc này ta chỉ cần chứng minh 

$\frac{a}{5a-1}\geq \frac{b}{16-5b}+\frac{c}{16-5c}$

Mà $\frac{b}{16-5b}+\frac{c}{16-5c}\leq \frac{b}{16-5a}+\frac{c}{16-5a}= \frac{3-a}{16-5a}\leq \frac{a}{5a-1}\Leftrightarrow \frac{1}{(5a-1)(16-5a)}> 0$

ĐÚng theo giả thiết,từ đây ta suy ra đ.p.c.m

Đẳng thức xảy ra khi và chỉ khi a=b=1/2 ,c=2 cùng hoán vị

 

Cách 2: 

Ta có BĐT tương đương

f(abc,a+b+c,ab+bc+ca) =$8.\frac{ab+bc+ca}{abc}+9-10((a+b+c)^{2}-2(ab+bc+ca))\geq 0$

Nhận thấy đây là hàm đơn điệu trên R theo abc nên theo định lý ABC, hàm số đạt cực tiểu khi có 2 biến = nhau, nên a=b=$\frac{3-c}{2}$

Thay vào và chứng minh BĐT 1 biến c ,bài toán trở nên quá đơn giản 

 

 

 

 


Bài 18:(caybutbixanh)Cho $x;y;z> 0$.Chứng minh rằng :

$P=\frac{2xy}{(z+x)(z+y)}+\frac{2yz}{(x+y)(x+z)}+\frac{3xz}{(y+z)(y+x)}\geqslant \frac{5}{3}.$

 

(trích đề thi học sinh giỏi lớp 11-Quảng Bình 2011)

--------------------------------

 

 

 

(T M) Hướng giải:

 

Bằng khai triển trực tiếp ta đưa bất đẳng thức cần chứng minh thành

 

$$xy(x+y)+yz(y+z)+4xz\left ( x+z \right )\geq 10xyz$$

 

Điều này tương đương với

 

$$\frac{x+y}{z}+\frac{y+z}{x}+\frac{4(x+z)}{y}\geq 10$$

 

Áp dụng $AM-GM$ từng cặp là ra.

 

 


Bài 19:(supermath98)Cho các sô dương $a;b;c$ thỏa mãn $\large ab+ac+bc=3abc$. Tìm GTNN của biểu thức: 

 

$\large M=\frac{2\left ( a^{2}b^{2}+b^{2}c^{2}+a^{2}c^{2} \right )+abc}{a^{2}b^{2}c^{2}}$

 

 

Bài giải:(thanhdok14)

 

 

Vì $a, b, c>0$ nên từ điều kiện ban đầu, ta suy ra:

$\frac{1}{a}+\frac{1}{b}+\frac{1}{c}=3$

Đặt: $\left(\frac{1}{a};\frac{1}{b};\frac{1}{c}\right)\to (x;y;z)$

$\Rightarrow x+y+z=3$

$\Rightarrow xy+yz+zx\le 3$

Mặt khác: $M$ được viết lại thành:

$M=2\left(\frac{1}{a^2}+\frac{1}{b^2}+\frac{1}{c^2}\right)+\frac{1}{abc}$

$=2(x^2+y^2+z^2)+xyz$

Lại có: $x^2+y^2+z^2=9-2(xy+yz+zx)$

$xyz\ge \frac{(x+y+z)[4(xy+yz+zx)-(x+y+z)^2]}{9}=\frac{4(xy+yz+zx)-9}{3}$   (theo $schur$)

Từ đó ta có:

$M\ge \frac{4}{3}(xy+yz+zx)-4(xy+yz+zx)+15=\frac{-8}{3}(xy+yz+zx)+15\ge 7$   (vì $xy+yz+zx\le 3$)

Vậy $min M=7\Leftrightarrow a=b=c=1$

 

 


Bài 20:(duaconcuachua)

Cho $a,b,c$ là các số thực dương thỏa mãn $ab+bc+ca=abc$.

Chứng minh rằng $\frac{a^{4}+b^{4}}{ab(a^{3}+b^{3})}+\frac{b^{4}+c^{4}}{bc(b^{3}+c^{3})}+\frac{c^{4}+a^{4}}{ca(c^{3}+a^{3})}\geq 1$

 Bài giải:(Sagittius912)Theo bđt Chebyshev ta có

 

 

$\frac{a^4+b^4}{a^3+b^3}\ge \frac{a+b}{2}$

do đó

 

$\frac{a^{4}+b^{4}}{ab(a^{3}+b^{3})}+\frac{b^{4}+c^{4}}{bc(b^{3}+c^{3})}+\frac{c^{4}+a^{4}}{ca(c^{3}+a^{3})}\ge \frac{a+b}{2ab}+\frac{b+c}{2bc}+\frac{c+a}{2ca}=\frac{ab+bc+ca}{abc}=1$

 

Dấu đẳng thức xảy ra khi $a=b=c=3$

 

 


 

 

 

 

 


Bài viết đã được chỉnh sửa nội dung bởi phanquockhanh: 23-07-2013 - 19:59


#2
CD13

CD13

    Thượng úy

  • Thành viên
  • 1456 Bài viết

Bài 21:

Cho các số thực dương $a,b,c$ thoả mãn:

 

                             $\frac{2}{a^{2}+1}+\frac{2}{b^{2}+1}+\frac{2}{c^{2}+1}\geq 3$

 

Chứng minh rằng: $(a-2)^{2}+(b-2)^{2}+(c-2)^{2}\geq 3$

Hướng dẫn:

$\frac{2}{a^{2}+1}+\frac{2}{b^{2}+1}+\frac{2}{c^{2}+1}\geq 3\Leftrightarrow \sum \frac{(1-a)(1+a)}{a^2+1}\geqslant 0$

$(a-2)^{2}+(b-2)^{2}+(c-2)^{2}-3=\sum (a-3)(a-1)$

Ta có :$\sum (a-3)(a-1)-\sum \frac{2(1-a)(1+a)}{a^2+1}=\sum \frac{(a-1)^4}{a^2+1}\geqslant 0$

           $\Leftrightarrow \sum (a-3)(a-1)\geqslant \sum \frac{2(1-a)(1+a)}{a^2+1}\geqslant 0$

           $\Leftrightarrow (a-2)^{2}+(b-2)^{2}+(c-2)^{2}\geq 3$

 

 

 

Bài 22:

Cho a,b,c>0 thỏa mãn abc=1

Chứng Minh : $2(\sum a^{2})+12\geq 3(\sum a)+3(\sum ab)$

Hướng dẫn:

Do $abc=1$ nên bất dẳng thức cần chứng minh có thể viết lại dưới dạng 

$\sum (2a^{2}+4-3a-\frac{3}{a})\geq 0$

 

Không mất tính tổng quát, giả sử $a=min{a,b,c}$. Dễ thấy $a\leq 1$ và $bc\geq 1$

Đặt $f(t)=2t^{2}+4-3t-\frac{3}{t}$ với $t>0$. Ta cần chứng minh $f(a)+f(b)+f$($c$)$\geq 0$

Trước hết ta sẽ chứng minh 

$f(b)+f$($c$)$\geq 2f(\sqrt{bc})$

$\Leftrightarrow 2b^{2}+2c^{2}-3b-3c-\frac{3}{b}-\frac{3}{c}\geq 4bc-6\sqrt{bc}-\frac{6}{\sqrt{bc}}$

$\Leftrightarrow 2(b-c)^{2}-3(\sqrt{b}-\sqrt{c})^{2}-3(\frac{1}{\sqrt{b}}-\frac{1}{\sqrt{c}})\geq 0$

$\Leftrightarrow (\sqrt{b}-\sqrt{c})^{2}(2b+2c+4\sqrt{bc}-3a-3)\geq 0$

Do $2b+2c\geq 3a$ và $\sqrt{bc}\geq 1$ nên $(2b+2c+4\sqrt{bc}-3a-3)\geq 0$

Suy ra $f(b)+f$($c$)$\geq 2f(\sqrt{bc})$

Đặt $\sqrt{a}=x$ với $x\leq 1$, ta cần chứng minh 

$f(x^{2})+2f(\frac{1}{x})\geq 0$

$\Leftrightarrow (2x^{4}+4-3x^{2}-\frac{3}{x^{2}})+2(\frac{4}{x^{2}}+8-\frac{6}{x}-6x)\geq 0$

$\Leftrightarrow 2x^{6}-3x^{4}-6x^{3}+12x^{2}-6x+1\geq 0$

$\Leftrightarrow (x-1)^{2}(2x^{4}+4x^{3}+3x^{2}-4x+1)\geq 0$

Do $2x^{4}+4x^{3}+3x^{2}-4x+1=x^{2}(x+1)^{2}+(x^{2}+2x-1)^{2}\geq 0$

nên $(x-1)^{2}(2x^{4}+4x^{3}+3x^{2}-4x+1)\geq 0$

Vậy $f(a)+f(b)+f$($c$)$\geq 0$

dấu đẳng thức xảy ra khi và chỉ khi $a=b=c=1$

Cách khác:

Bđt tương đương: $2(a+b+c)+12\geq 3(a+b+c)+7|(ab+bc+ac)$

đặt : $a+b+c=p,ab+bc+ca=q \Rightarrow p^{3}+9r\geq 4pq$ (Schur)

Nên: Ta chỉ cần chứng minh: $2P^{2}+12\geq 3P+7(\frac{p^{3}+9}{4p})\Leftrightarrow (p-3)(p^{2}-9p+21)\geq 0$

điều này luôn đúng vì p$\geq$3

 

 

 

Bài 23:

Ccho a,b,c la cac số thực không âm có tổng bằng 1. CMR:

$\frac{a}{\sqrt[3]{a+2b}}+\frac{b}{\sqrt[3]{b+2c}}+\frac{c}{\sqrt[3]{c+2a}}\geq 1$

Hướng dẫn:

Đặt S=$\sum \frac{a}{\sqrt[3]{a+2b}}$

P=$\sum a(a+2b)$

Áp dung BDT holder cho 4cặp số $\Rightarrow S^{3}P\geq (a+b+c+d)^{4}\Rightarrow S^{3}\geq \frac{(a+b+c+d)^{4}}{\sum a(a+2b)}= (a+b+c+d)^{2}= 1\Rightarrow S\geq 1$ (dpcm)

Cách khác:

$\sum \frac{a}{\sqrt[3]{a+2b}}\geq \frac{3a}{a+2b+2}\geq \frac{3(a+b+c)^2}{a^2+b^2+c^2+2(ab+ac+bc)+2(a+b+c)}=1$

Cách khác:

Ta có $\sqrt[3]{1.1.(a+2b)}\leq \frac{a+2b+2}{3}$

$VT=\sum \frac{a^{2}}{\sqrt[3]{a+2b}}\geq \sum \frac{3a^{2}}{a+2b+2}=3(\sum \frac{a^{2}}{a^{2}+2ab+2a})\geq 3\frac{(a+b+c)^{2}}{(a+b+c)^{2}+2(a+b+c)}=3.\frac{1}{3}=1.$

Dấu $"="$ xảy ra $\Leftrightarrow a=b=c=\frac{1}{3}$

 

 

 

Bài 24:

Cho $a,b,c\in\mathbb{R^+}$ sao cho $a+b+c=4$. Bất đẳng thức sau luôn đúng :

\[ 10\left(\frac{b}{a}+\frac{c}{b}+\frac{a}{c}\right) \ge \frac{4+5a}{4-a}+\frac{4+5b}{4-b}+\frac{4+5c}{4-c} \]

Hướng dẫn:

Bài toán phụ : 

Cho các số thực dương $a$,$b$,$c$. Khi đó, bất đẳng thức sau luôn đúng : 

$\frac{a}{b}+\frac{b}{c}+\frac{c}{a}\geq \frac{a+c}{b+c}+\frac{b+a}{c+a}+\frac{c+b}{a+b}$

Chứng minh :

Sử dụng các hằng đẳng thức sau : 

$\frac{a}{b}+\frac{b}{c}+\frac{c}{a}-3=\frac{(a-b)^{2}}{ab}+\frac{(a-c)(b-c)}{ac}$

 

$\frac{a+c}{b+c}+\frac{b+a}{c+a}+\frac{c+b}{a+b}-3=\frac{(a-b)^{2}}{(a+c)(b+c)}+\frac{(a-c)(b-c)}{(a+c)(a+b)}$

Do đó, bất đẳng thức cần chứng minh trở thành 

$[\frac{1}{ab}-\frac{1}{(a+c)(b+c)}](a-b)^{2}+[\frac{1}{ac}-\frac{1}{(a+c)(a+b)}](a-c)(b-c)\geq 0$

 

Giả sử $c=min${$a$,$b$,$c$}, ta được đpcm.

Trở lại bài toán, bất đẳng thức cần chứng minh tương đương với 

$10\sum \frac{b}{a}\geq \sum \frac{4+5a}{4-a}=\sum (\frac{6a}{b+c}+1)=\sum \frac{6a}{b+c}+3$

Áp dụng bài toán phụ trên, ta có : 

$\sum \frac{b}{a}\geq \sum \frac{b+c}{a+c}>\sum \frac{b}{a+c}\Rightarrow 6\sum \frac{b}{a}>6\sum \frac{b}{a+c}$

Theo AM-GM : $\sum \frac{b}{a}\geq 3$

Do đó : $7\sum \frac{b}{a}> \sum \frac{6a}{b+c}+3$

Bài toán này có một mở rộng khá hay, đó là 

$2\sum \frac{a}{b}+2\sum \frac{b}{a}\geq 6\sum \frac{a}{b+c}+3$

Không biết kết quả trên có đúng không, chỉ biết nếu sử dụng bài toán phụ trên $2$ lần thì lại được kết quả yếu hơn :

$2\sum \frac{a}{b}+2\sum \frac{b}{a}\geq 4\sum \frac{a}{b+c}+6$

 

 

 

Bài 25:

Cho $a,b,c\in\mathbb{R^+}$ và $a+b+c=\sqrt[3]{2}$. Các em học sinh không được dùng dồn biến, hãy chứng minh rằng:

\[ \dfrac{a^2}{4a+1}+\dfrac{b^2}{4b+1}+\dfrac{c^2}{4c+1}\le \dfrac{1}{32(ab+bc+ac)}+\dfrac{7-2\sqrt[3]{2}}{16\sqrt[3]{4}+4\sqrt[3]{2}} \]

Hướng dẫn:

Bài này chúng ta có thể đặt $a+b+c=\sqrt[3]{2}=x$ cho dễ nhìn :))

Ta có : 

$\frac{a}{4}-\frac{a^{2}}{4a+1}=\frac{a}{4(4a+1)}$

$\Rightarrow \frac{x}{4}-\sum \frac{a^{2}}{4a+1}=\sum \frac{a}{4(4a+1)}$

Áp dụng $Cauchy-Schwarz$, ta có :

$\sum \frac{a}{4a+1}\geq \frac{(a+b+c)^{2}}{\sum 4a^{2}+(a+b+c)}=\frac{x^{2}}{\sum 4a^{2}+x}$

Bất đẳng thức cần chứng minh tương đương với 

$\frac{x^{2}}{4(\sum 4a^{2}+x)}+\frac{1}{32\sum ab}+\frac{7-2x}{16x^{2}+4x}\geq \frac{x}{4}$

$\Leftrightarrow \frac{x^{2}}{4(\sum 4a^{2}+x)}+\frac{1}{32\sum ab}\geq \frac{x^{2}+2x+1}{4(4x^{2}+x)}$

$\Leftrightarrow \frac{x^{2}}{\sum 4a^{2}+x}+\frac{1}{8\sum ab}\geq \frac{(x+1)^{2}}{4x^{2}+x}$

 

Áp dụng $Cauchy-Schwarz$, ta có :

$\frac{x^{2}}{\sum 4a^{2}+x}+\frac{1}{8\sum ab}\geq \frac{(x+1)^{2}}{\sum 4a^{2}+8\sum ab+x}=\frac{(x+1)^{2}}{4x^{2}+x}$

Bài toán chứng minh xong. Dấu đẳng thức xảy ra khi và chỉ khi $a=b=c=\frac{x}{3}=\frac{\sqrt[3]{2}}{3}$

 

 

 

Bài 26:

Cho $x,y,z$ là các số thực thuộc $[0;1]$. Tìm giá trị lớn nhất của biểu thức:

$P=\frac{x}{1+yz}+\frac{y}{1+zx}+\frac{z}{1+xy}$

Hướng dẫn:

Ta có:

Không mất tính tổng quát,giả sử $x\geq y\geq z$$\Rightarrow \sum \frac{x}{1+yz}\leq \frac{x+y+z}{1+yz}\leq 2\Leftrightarrow 2+2yz\geq x+y+z\Leftrightarrow (1+yz-x)+(y-1)(z-1)\geq 0$

Hiển nhiên đúng theo giả thiết

Ta có đ.p.c.m

Max P =2 khi x=y=1,z=0 cùng hoán vị

Cách khác:

$x,y,z \in [0;1]$ $\Rightarrow x(y-1)(z-1)+(x-1)(y+z-2) \geq 0$ $\Rightarrow x+y+z \leq xyz+2 \leq 2xyz+2$

$P=\dfrac{x}{1+yz}+\dfrac{y}{1+zx}+\dfrac{z}{1+xy} \leq \dfrac{x+y+z}{1+xyz} \leq \dfrac{2+2xyz}{1+xyz}=2$

Vậy $P_{max}=2$ khi $x=y=1, z=0$.

 

 

 

Bài 27:

Cho $a,b,c,d >0$ và $a+b+c+d=4$.

Tìm GTNN của $P=(a^3+4)(b^3+4)(c^3+4)(d^3+4)$

Hướng dẫn

Áp dụng BDT Holder ta có được : 

$(a^3+4)(b^3+4)(c^3+4)\geq (a+b+c+2)^3=(6-d)^3$

 

Từ đây :

$P\geq (6-d)^3(d^3+4)=-(d-1)^2(d^4-16d^3+75d^2-46d-239)+5^4$

 

Nếu như giả sử : $d=min(a,b,c,d)$ thì $d\leq 1$

 

Hiển nhiên : $-(d-1)^2(d^4-16d^3+75d^2-46d-239)\geq 0$

 

Vậy nên : $P\geq 5^4$

 

Dấu bằng tại $a=b=c=d=1$

 

 

 

Bài 28:

Cho các số thực duơng $a,b,c,x,y,z$ thoả mãn: $ab+bc+ca=xy+yz+zx=1$. Chứng minh rằng:

                            $a(y+z) +b(z+x) + c(x+y) \geq 2$

Hướng dẫn:

Ta có $a(y+z)+b(z+x)+c(x+y) \geq 2$

$\Leftrightarrow ax+by+cz+2\leq (a+b+c)(x+y+z)$.

Và $(ax+by+cz)^{2}\leq (a^{2}+b^{2}+c^{2})(x^{2}+y^{2}+z^{2})$

$\Rightarrow (ax+by+cz)\leq \sqrt{(a^{2}+b^{2}+c^{2})(x^{2}+y^{2}+z^{2})}$.

Nên $\Rightarrow (ax+by+cz)+2\leq \sqrt{(a^{2}+b^{2}+c^{2})(x^{2}+y^{2}+z^{2})}+2$

Và $\sqrt{(a^{2}+b^{2}+c^{2})(x^{2}+y^{2}+z^{2})}+2=\sqrt{(a^{2}+b^{2}+c^{2})(x^{2}+y^{2}+z^{2})}+\sqrt{2.2}$

$\leq \sqrt{(a^{2}+b^{2}+c^{2}+2)(x^{2}+y^{2}+z^{2}+2)}$.

Đến đây,với $ab+bc+ca=xy+yz+zx=1$ thì hiển nhiên ta có điều phải chứng minh.

 

 

 

Bài 29:

Cho $\dfrac{1}{3} < x \le \dfrac{1}{2}$ và $y \ge 1$
Tìm GTNN của biểu thức:
$$P=x^2+y^2+\dfrac{x^2y^2}{[(4x-1)y-x]^2}$$

Hướng dẫn:

Đặt $a=\frac{1}{x},b=\frac{1}{y}$ thì $2\leq a<3$ và $0<b\leq 1$

 

Khi đó: $P=\frac{1}{a^{2}}+\frac{1}{b^{2}}+\frac{1}{(4-a-b)^{2}}$

 

Áp dụng bất đẳng thức Cauchy cho hai số dương ta có:

 

$P\geq \frac{1}{a^{2}}+\frac{2}{b(4-a-b)}\geq \frac{1}{a^{2}}+\frac{1}{(4-a)^{2}}+\frac{7}{(4-a)^{2}}\geq \frac{2}{a(4-a)}+\frac{7}{(4-a)^{2}}\geq \frac{9}{4}$ (do $a \geq 2$)

 

Đẳng thức xảy ra khi và chỉ khi $a=2,b=1$, hay $x= \frac{1}{2}, y=1$

 

Vậy $minP= \frac{9}{4}$

 

 

 

Bài 30:

Cho các số thực dương $a$,$b$,$c$ thoả mãn $a+b+c=3$.

Chứng minh : $\frac{1}{a^{2}+b^{2}+2}+\frac{1}{b^{2}+c^{2}+2}+\frac{1}{c^{2}+a^{2}+2}\leq \frac{3}{4}$

Hướng dẫn:

BĐT đã cho tương đương với 

$$\frac{1}{2}-\frac{1}{a^2+b^2+2}+\frac{1}{2}-\frac{1}{b^2+c^2+2}+\frac{1}{2}-\frac{1}{c^2+a^2+2}\geq \frac{3}{4}$$

$$\Leftrightarrow \frac{a^2+b^2}{a^2+b^2+2}+\frac{b^2+c^2}{b^2+c^2+2}+\frac{c^2+a^2}{c^2+a^2+2}\geq \frac{3}{2}$$

$$\Leftrightarrow \sum \frac{(a+b)^2}{2(a^2+b^2+2)}+\sum \frac{(a-b)^2}{2(a^2+b^2+2)}\geq \frac{3}{2}$$

Áp dụng BĐTCauchy-Schwarz, ta có:

$$\sum \frac{(a+b)^2}{2(a^2+b^2+2)}\geq \frac{4(a+b+c)^2}{4(a^2+b^2+c^2)+12}=\frac{2(a+b+c)^2}{2(a^2+b^2+c^2)+6}$$

$$\sum \frac{(a-b)^2}{2(a^2+b^2+2)}\geq \frac{[(a-b)+(b-c)+(a-c)]^2}{4(a^2+b^2+c^2)+12}=\frac{2(a-c)^2}{2(a^2+b^2+c^2)+6}$$

Cộng 2 BĐT này lại, ta có

$$VT\geq \frac{2(a+b+c)^2+2(a-c)^2}{2(a^2+b^2+c^2)+6}$$

Do đó ta chỉ cần chứng minh

$$2(a+b+c)^2+2(a-c)^2\geq 3(a^2+b^2+c^2)+9=3(a^2+b^2+c^2)+(a+b+c)^2$$

Bằng cách khai triển, ta được BĐT này tương đương với

$$2(a-b)(b-c)\geq 0$$

BĐT hiển nhiên đúng nếu ta giả sử $a\geq b\geq c$

Kết thúc chứng minh

Cách khác:

Ta có:

BĐT tương đương 

$\sum \frac{(a+b)^{2}}{(a+b)^{2}+\frac{2(a+b)^{2}}{a^{2}+b^{2}}}\geq \frac{3}{2}$

Áp dụng BCS ,ta cần chứng minh

:$\sum (a+b)^{2}+\sum \frac{2(a+b)^{2}}{a^{2}+b^{2}}\leq 24\Leftrightarrow \sum (a-b)^{2}(\frac{1}{a^{2}+b^{2}}-\frac{1}{6})\geq 0$

Giả sử $a\geq b\geq c$

Nếu $a^{2}+b^{2}\leq 6\Rightarrow \blacksquare$

Nếu $a^{2}+b^{2}\leq 6\Rightarrow\sum \frac{1}{a^{2}+b^{2}+2}\leq \frac{1}{8}+\frac{1}{8}+\frac{1}{2}= \frac{3}{4}\Rightarrow \blacksquare$


Bài viết đã được chỉnh sửa nội dung bởi phanquockhanh: 23-07-2013 - 20:06


#3
CD13

CD13

    Thượng úy

  • Thành viên
  • 1456 Bài viết

Bài 31:

Cho x,y,z $\geq$ 0 thỏa mãn $x^{2}+y^{2}+z^{2}=1$. Tìm MAX:

  $P=6(-x+y+z)+27xyz$

Hướng dẫn:

Ta có:

$1-x^2=y^2+z^2\geq \frac{1}{2}(y+z)^2 \Rightarrow y+z \leq \sqrt{2-2x^2}$ 

$yz \leq \dfrac{y^2+z^2}{2}=\dfrac{1-x^2}{2}$

Nên suy ra:

$P \leq 6(-x+\sqrt{2-2x^2})+ \dfrac{27}{2}x(1-x^2)$

Xét hàm số: 

$f(x)=6(-x+\sqrt{2-2x^2})+ \dfrac{27}{2}x(1-x^2)$, với $0\leq x \leq 1$

Ta có : 

$f'(x)=6\left ( -1-\dfrac{2x}{\sqrt{2-2x^2}} \right )+ \dfrac{27}{2}(1-3x^2)$

Ta thấy $x=\frac{1}{3}$ là một nghiệm của $f'(x)=0$

mà $f''(x) <0$ với $0 \leq x < 1$ nên $f'(x)=0$ chỉ có một nghiệm duy nhất $x=\frac{1}{3}$ trên nửa khoảng xét

lập bảng biến thiên ta được:

$f(x) \leq f( \frac{1}{3}) =10$

Vậy $MaxP=10$ khi $x= \frac{1}{3}, y=z= \frac{2}{3}$

 

 

 

Bài 32:

Cho $a,b,c$ là các số thực dương thỏa mãn $ab+bc+ca=abc$.

Chứng minh rằng $\frac{a^{4}+b^{4}}{ab(a^{3}+b^{3})}+\frac{b^{4}+c^{4}}{bc(b^{3}+c^{3})}+\frac{c^{4}+a^{4}}{ca(c^{3}+a^{3})}\geq 1$

Hướng dẫn:

Theo bđt Chebyshev ta có

 

$\frac{a^4+b^4}{a^3+b^3}\ge \frac{a+b}{2}$

do đó

 

$\frac{a^{4}+b^{4}}{ab(a^{3}+b^{3})}+\frac{b^{4}+c^{4}}{bc(b^{3}+c^{3})}+\frac{c^{4}+a^{4}}{ca(c^{3}+a^{3})}\ge \frac{a+b}{2ab}+\frac{b+c}{2bc}+\frac{c+a}{2ca}=\frac{ab+bc+ca}{abc}=1$

 

Dấu đẳng thức xảy ra khi $a=b=c=3$

 

 

 

Bài 33:

Cho $x+y=2$. Tìm MAX của $A= (x^3+2)(y^3+2)$

Hướng dẫn:

Đặt $S = x + y = 2; P = xy \leq \dfrac{S^2}{4} = 1$
Ta có:
$A = (x^3 + 2)(y^3 + 2) = (xy)^3 + 2(x^3 + y^3) + 4$
$A = P^3 + 2(S^3 - 3PS) + 20 = P^3 - 12P + 20$
Xét hàm số $f(t) = t^3 - 12t + 20$ trên $(- \propto; 1]$ có đạo hàm $f’(t) = 3t^2 - 12$
Ta thấy: $f'(t) = 0 \Leftrightarrow t = \pm 2 $
  • $f’(t) > 0$ $\forall$ $t \in (- \propto; - 2)$
  • $f’(t) < 0$ $\forall$ $t \in (-2; 1]$
Vì vậy: $Max_{A} = 36$ khi $P = - 2 \Rightarrow x = 1+ \sqrt{3}; y = 1 - \sqrt{3}$
 
 
Bài 34:
Cho $a,b,c> 0$.CMR:

          $\frac{a^{3}}{b^{2}-bc+c^{2}}+\frac{b^{3}}{c^{2}-ca+a^{2}}+\frac{c^{3}}{a^{2}-ab+b^{2}}\geq \frac{3(ab+bc+ca)}{a+b+c}$

Hướng dẫn:

Ta có $\frac{a^{3}}{b^{2}-bc+c^{2}}+b+c= \frac{a^{3}+b^{3}+c^{3}}{b^{2}-bc+c^{2}}$

Áp dụng BDT co si dạng engel $\Rightarrow \sum \frac{a^{3}+b^{3}+c^{3}}{a^{2}-ab+b^{2}}\geq \frac{9(a^{3}+b^{3}+c^{3})}{2(a^{2}+b^{2}+c^{2})-ab-bc-ca }$

Lai có $\left ( a+b+c \right )^{2}\geq 3(ab+bc+ca)\Rightarrow a+b+c\geq \frac{3(ab+bc+ca)}{a+b+c}$

Ta chỉ viec CM $\frac{9(a^{3}+b^{3}+c^{3})}{2(a^{2}+b^{2}+c^{2})-ab-bc-ca}\geq a+b+c$

Biến đổi tương đương ta đc $a^{3}+b^{3}+c^{3}+3abc\geq \sum (ab(a+b))$(luôn đúng theo BDT schur)

 

 

 

Bài 35:

Cho các số dương $a;b;c$ thỏa mãn $\large ab+ac+bc=3abc$. Tìm GTNN của biểu thức: 

$\large M=\frac{2\left ( a^{2}b^{2}+b^{2}c^{2}+a^{2}c^{2} \right )+abc}{a^{2}b^{2}c^{2}}$

Hướng dẫn:

Vì $a, b, c>0$ nên từ điều kiện ban đầu, ta suy ra:

$\frac{1}{a}+\frac{1}{b}+\frac{1}{c}=3$

Đặt: $\left(\frac{1}{a};\frac{1}{b};\frac{1}{c}\right)\to (x;y;z)$

$\Rightarrow x+y+z=3$

$\Rightarrow xy+yz+zx\le 3$

Mặt khác: $M$ được viết lại thành:

$M=2\left(\frac{1}{a^2}+\frac{1}{b^2}+\frac{1}{c^2}\right)+\frac{1}{abc}$

$=2(x^2+y^2+z^2)+xyz$

Lại có: $x^2+y^2+z^2=9-2(xy+yz+zx)$

$xyz\ge \frac{(x+y+z)[4(xy+yz+zx)-(x+y+z)^2]}{9}=\frac{4(xy+yz+zx)-9}{3}$   (theo $schur$)

Từ đó ta có:

$M\ge \frac{4}{3}(xy+yz+zx)-4(xy+yz+zx)+15=\frac{-8}{3}(xy+yz+zx)+15\ge 7$   (vì $xy+yz+zx\le 3$)

Vậy $min M=7\Leftrightarrow a=b=c=1$

 

 

 

Bài 36:

Cho $a,b,c >0$ và $a+b+c=3$

Chứng minh rằng : $8(\frac{1}{a}+\frac{1}{b}+\frac{1}{c})+9 \geq 10(a^2+b^2+c^2)$

Hướng dẫn:

Ta có:

BĐT tương đương

$\frac{5}{4}(\frac{1}{a}+\frac{1}{b})-10(a^{2}+b^{2})+\frac{27}{4}(\frac{1}{a}+\frac{1}{b})+\frac{8}{c}+9-10c^{2}\geq 0\Leftrightarrow 5\frac{(a+b)-8ab(a^{2}+b^{2})}{4ab}+\frac{27}{4}(\frac{1}{a}+\frac{1}{b})+\frac{8}{c}+9-10c^{2}\geq 0$

Lại có : $4ab\leq (a+b)^{2}$ , $8ab(a^{2}+b^{2})\leq (a+b)^{4}$,$\frac{1}{a}+\frac{1}{b}\geq \frac{4}{a+b}$

Nên VT $\geq A= 5.\frac{1-(a+b)^{3}}{a+b}+\frac{27}{a+b}+\frac{8}{c}+9-10c^{2}$

Thay a+b=3-c,ta thấy $\geq 0\Leftrightarrow (2-c)\frac{(35c^{2}+12-6c-15c^{3})}{c(3-c)}\geq 0$

DO vai trò của a,b,c như nhau,không mất tính tổng quát,giả sử c không lớn hơn 2, nên 2-c không âm

Còn phương trình $35c^{2}+12-6c-15c^{3}$ ,ta nhận thấy,theo như điều giả sử thì $15c^{3}\leq 30c^{2}$ và $6c\leq 12$ nên hiển nhiên biểu thức đó lớn hơn 0,từ đây hoàn tất cm ban đầu

Như vậy,ta đã chứng minh được bài toán.

Đẳng thức xảy ra khi và chỉ khi $a=b=\frac{1}{2}$ , $c=2$ cùng các hoán vị

Cách khác:

Cách 1

Giải như sau:

Giả sử a là số lớn nhất trong 3 số a,b,c ,thế thì c nhỏ hơn 3 và không nhỏ hơn 1

ta thấy $9=(42a-48)+(42b-\frac{69}{2})+(42c-\frac{69}{2})$

Thay và BĐT ban đầu ta thấy tương đương

$(\frac{8}{b}-10b^{2}+42b-\frac{69}{2})+(\frac{8}{c}-10c^{2}+42c-\frac{69}{2})\geq 10a^{2}-\frac{8}{a}-42a+48\Leftrightarrow \frac{(16-5b)(2b-1)^{2}}{b}+\frac{(16-52)(2c-1)^{2}}{c}\geq \frac{4(5a-1)(a-2)^{2}}{a}$

Áp dụng BCS ,ta có:

VT $\geq \frac{(2b-1+2c-1)^{2}}{\frac{b}{16-5b}+\frac{c}{16-5c}}= \frac{4(a-2)^{2}}{\frac{b}{16-5b}+\frac{c}{16-5c}}$

Lúc này ta chỉ cần chứng minh 

$\frac{a}{5a-1}\geq \frac{b}{16-5b}+\frac{c}{16-5c}$

Mà $\frac{b}{16-5b}+\frac{c}{16-5c}\leq \frac{b}{16-5a}+\frac{c}{16-5a}= \frac{3-a}{16-5a}\leq \frac{a}{5a-1}\Leftrightarrow \frac{1}{(5a-1)(16-5a)}> 0$

ĐÚng theo giả thiết,từ đây ta suy ra đ.p.c.m

Đẳng thức xảy ra khi và chỉ khi a=b=1/2 ,c=2 cùng hoán vị

Cách 2: 

Ta có BĐT tương đương

f(abc,a+b+c,ab+bc+ca) =$8.\frac{ab+bc+ca}{abc}+9-10((a+b+c)^{2}-2(ab+bc+ca))\geq 0$

Nhận thấy đây là hàm đơn điệu trên R theo abc nên theo định lý ABC, hàm số đạt cực tiểu khi có 2 biến = nhau, nên a=b=$\frac{3-c}{2}$

Thay vào và chứng minh BĐT 1 biến c ,bài toán trở nên quá đơn giản

 

 

Bài 37:

Cho các số thực duơng $a,b,c$ thoả mãn: $ab+bc+ca\leq 3abc$. Chứng minh rằng:

$\sqrt{\frac{a^{2}+b^{2}}{a+b}}+\sqrt{\frac{b^{2}+c^{2}}{b+c}}+\sqrt{\frac{c^{2}+a^{2}}{c+a}}+3\leq \sqrt{2}(\sqrt{a+b}+\sqrt{b+c}+\sqrt{c+a})$

Hướng dẫn:

Áp dụng bất đẳng thức $\sqrt{2(u+v)}\geq \sqrt{u}+\sqrt{v},\forall u,v\geq 0$, ta có

$VT=\sum \sqrt{2(a+b)}=\sqrt{2}\sum\sqrt{\frac{a^{2}+b^{2}}{a+b}+\frac{2ab}{a+b}}$

$\geq \sum \sqrt{\frac{a^{2}+b^{2}}{a+b}}+\sum \sqrt{\frac{2}{\frac{1}{a}+\frac{1}{b}}}$

Do đó ta chỉ cần chứng minh

$\sum \sqrt{\frac{2}{\frac{1}{a}+\frac{1}{b}}}\geq 3$

Đặt $(x,y,z)=\left ( \frac{1}{a},\frac{1}{b},\frac{1}{c} \right )$ với $x,y,z>0, x+y+z\leq 3$

Ta có

$\sum \sqrt{\frac{2}{x+y}}\geq \frac{9\sqrt{2}}{\sum \sqrt{x+y}} \geq \frac{9\sqrt{2}}{\sqrt{3\sum (x+y)}}\geq \frac{9\sqrt{2}}{\sqrt{3.2.3}}=3$

Từ đó suy ra

$\sum \sqrt{2(a+b)}\geq \sum \sqrt{\frac{a^{2}+b^{2}}{a+b}}+3$

Dấu "=" xảy ra $\iff a=b=c=1$

 

 

 

Bài 38:

Cho các số thực dương a,b,c CMR $\sum \frac{a^{2}+b^{2}}{a^{2}+ab+b^{2}}\leq \frac{6(a^{2}+b^{2}+c^{2})}{(a+b+c)^{2}}$

Hướng dẫn:

Ta có $\sum \frac{a^2+b^2}{a^2+ab+b^2} = 6- \sum \frac{(a+b)^2}{a^2+ab+b^2}$. Do đó bài toán đưa về việc chứng minh $$\sum \frac{(a+b)^2}{a^2+ab+b^2}+ \dfrac{6(a^2+b^2+c^2)}{(a+b+c)^2} \ge 6$$

Áp dụng BĐT Cauchy-Schwarz ta có $$\sum \frac{(a+b)^2}{a^2+ab+b^2} \ge \dfrac{4(a+b+c)^2}{2 \sum a^2 + \sum ab}$$.

Ta chỉ cần chứng minh $$\dfrac{4(a+b+c)^2}{2 \sum a^2 + \sum ab} + \dfrac{6(a^2+b^2+c^2)}{(a+b+c)^2} \ge 6$$

Đặt $a^2+b^2+c^2=x,ab+bc+c=y$ thì $x \ge y$. Khi đó bất đẳng thức tương đương với $$\frac{4(x+2y)}{2x+y}+ \frac{6x}{x+2y} \ge 6 \Leftrightarrow \frac{4(x+2y)}{2x+y}+ \frac{4(2x+y)}{x+2y} \ge 8$$

Bất đẳng thức cuối hiển nhiên đúng theo AM-GM. 

Dấu đẳng thức xảy ra khi và chỉ khi $a=b=c$.

 

 

 

Bài 39:

Cho các số thực dương $a,b,c$ thoả mãn: $a^{3}+b^{3}+c^{3}=1$. Chứng minh rằng:

 

$\frac{1}{a^{5}(b^{2}+c^{2})^{2}}+\frac{1}{b^{5}(c^{2}+a^{2})^{2}}+\frac{1}{c^{5}(a^{2}+b^{2})^{2}}\geq \frac{81}{4}$

Hướng dẫn:

Áp dụng BĐT AM-GM ta có $$\begin{array}{l} \frac{1}{a^5(b^2+c^2)^2}+ \frac{81a(b^2+c^2)}{8}+ \frac{81a(b^2+c^2)}{8} \ge \frac{27 \sqrt[3]{9}}{4a} \qquad (1) \\ \frac{1}{b^5(a^2+c^2)^2} + \frac{81b(a^2+c^2)}{8}+ \frac{81b(a^2+c^2)}{8} \ge \frac{27 \sqrt[3]{9}}{4b} \qquad (2) \\ \frac{1}{c^5(a^2+b^2)^2}+ \frac{81c(a^2+b^2)}{8}+ \frac{81c(a^2+b^2)}{8} \ge \frac{27\sqrt[3]{9}}{4c} \qquad (3) \end{array}$$

Áp dụng tiếp BĐT AM-GM thì $\frac{27 \sqrt[3]{9}}{4a}= 81 \cdot \frac{1}{4 \cdot 3 \cdot a \cdot \frac{1}{ \sqrt[3]{3}} \cdot \frac{1}{ \sqrt[3]{3}}} \ge \dfrac{81}{4 \left( a^3+ \frac 23 \right)}$. Tương tự $\dfrac{27 \sqrt[3]{9}}{4b} \ge \frac{81}{4 \left( b^3+ \frac 23 \right)}, \; \dfrac{27 \sqrt[3]{9}}{4c} \ge \frac{81}{4 \left( c^3+ \frac 23 \right)}$.

Như vậy cộng ba BĐT trên rồi áp dụng Cauchy-Schwarz ta có $$\sum \frac{27 \sqrt[3]{9}}{4a} \ge \sum \frac{81}{4 \left( a^3+ \frac 23 \right)} \ge \frac{27^2}{4(a^3+b^3+c^3)+8}= \frac{243}{4}$$

Bây giờ cộng các vế của $(1),(2),(3)$ lại với nhau ta được $$\sum \dfrac{1}{a^5(b^2+c^2)^2} \ge \frac{243}{4}- \frac{81\left( \sum ab^2 + \sum a^2b \right)}{4}$$

Bây giờ chỉ cần chứng minh $$\sum ab^2+ \sum a^2b \le 2(a^3+b^3+c^3) \qquad (3)$$

Áp dụng BĐT AM-GM thì $$\left. \begin{array}{l} a^3+a^3+b^3 \ge 3a^2b \\ b^3+b^3+c^3 \ge 3b^2c \\ c^3+c^3+a^3 \ge 3c^2a \end{array} \right \} \Rightarrow a^3+b^3+c^3 \ge a^2b+b^2c+c^2a$$

Và $$\left. \begin{array}{l} a^3+b^3+b^3 \ge 3ab^2 \\ b^3+c^3+c^3 \ge 3bc^2 \\ c^3+a^3+a^3 \ge 3ca^2 \end{array} \right \} \Rightarrow a^3+b^3+c^3 \ge ab^2+bc^2+ca^2$$

Vậy $(3)$ được chứng minh hay $\sum a^2b+ \sum ab^2 \le 2$. Do đó $\sum \frac{1}{a^5(b^2+c^2)^2} \ge \frac{243-162}{4}= \frac{81}{4}$.

Dấu đẳng thức xảy ra khi và chỉ khi $a=b=c= \frac{1}{\sqrt[3]{3}}$.

 

 

 

Bài 40:

Cho a,b,c>0 thoả $a^{2}+b^{2}+c^{2}=1$

 

Cmr : $\sum \frac{a}{a^{3}+bc}\geq 3$

Hướng dẫn:

Ta sẽ đổi biến cái này 

$\frac{ab}{c}=z$ và tương tự ,thế thì xy+yz+xz =1

$\sum \frac{a}{a^{3}+bc}= \sum \frac{1}{x+yz}\geq 3\Leftrightarrow x+y+z+7xyz(x+y+z)\geq 3x^{2}y^{2}z^{2}+2+3xyz(x+y+z)$ (bằng quy đồng rồi khai triển)

Ta lại có : $\Leftrightarrow (x+y+z-2)(1+6xyz-3xyz(x+y+z))+xyz(x+y+z-3xyz)\geq 0$

Nếu x+y+z $\leq 2$ thì $\sum \frac{1}{x+yz}\geq \frac{9}{2+1}= 3$
 

Còn nếu $x+y+z\leq 2\Rightarrow 1+6xyz-3xyz(x+y+z)= (xy+yz+zx)^{2}-3zxy(x+y+z)+1> 0$ 

và $xy+yz+xz= 1\Rightarrow xyz(x+y+z)\geq 3x^{2}y^{2}z^{2}$

vậy trong cả 2 trường hợp của x+y+z ta đều có đ.p.c.m

Đẳng thức không xảy ra

Cách khác:

$A = \frac{a}{a^3+ab} +\frac{b}{b^3+ac} +\frac{c}{c^3+ab}$

 

$= \frac{1}{a^2+\frac{bc}{a}} +\frac{1}{b^2+\frac{ac}{b}} +\frac{1}{c^2+\frac{ab}{c}}$

 

Đặt $\frac{bc}{a} = x$ , $\frac{ac}{b} = y$ , $\frac{ab}{c} = z$ . ta có $xy+yz+xz=1$ và 

 

$A=\frac{1}{yz+x}+\frac{1}{xz+y}+ \frac{1}{xy+z}$

 

Dễ thấy 

 

$A \geq 3 \Leftrightarrow 3(x+yz)(y+xz)(z+xy) - \sum (x+yz)(y+xz)\leq 0$

 

Là một đa thức bậc $2$ theo $xyz$ với hệ số bậc cao nhất không âm nên đa thức đạt cực đại khi có hai biến bằng nhau hoặc một biến bằng $0$ .

 

T/h $1$ : giả sử $x=z$ biểu diễn $y$ theo $x$ rồi đưa về $1$ biến , khảo sát là được 

 

T/h $2$ : có $1$ biến bằng $0$ ( vô lí vì $a,b,c$ dương )

 

 

 

 

 



#4
phanquockhanh

phanquockhanh

    Sĩ quan

  • Thành viên
  • 310 Bài viết

Bài 100:(b2stfs)

Cho a,b,c >0 thoả mãn a+b+c=3.Tìm GTNN của:
P=$\sum \frac{b\sqrt{b}}{\sqrt{2a+b+c}}$

 

Bài giải:(Sagittrius912)

Ta có

$P^{2}.((2a+b+c)+(2b+c+a)+(2c+a+b))= \sum \frac{b\sqrt{b}}{\sqrt{2a+b+c}}.\sum \frac{b\sqrt{b}}{\sqrt{2a+b+c}}.((2a+b+c)+(2b+c+a)+(2c+a+b))\geq (a+b+c)^{3}$

(bdt Holderr)

$\Rightarrow P^{2}\geq \frac{(a+b+c)^{2}}{4}\Rightarrow P\geq \frac{3}{2}$

Vậy min P=3/2 khi a=b=c=1

 

 

Bài 101:(b2stts)

Cho a,b,c>0 thoả mãn:a+b+c=$\frac{1}{2}$,Tìm GTLN:
$\sum \sqrt{\frac{(a+b)(b+c)}{(a+b)(b+c)+a+c}}$

Bài giải:(Mazacar)

Đặt $x=a+b;y=b+c;z=c+a$
Có:
$P=\sum \sqrt{\frac{xy}{xy+z}}=\sum \sqrt{\frac{xy}{(x+z)(z+y)}}\leq\frac{1}{2}\sum (\frac{x}{x+z}+\frac{y}{z+y})=\frac{3}{2}\blacksquare$
$"=" <=>x=y=z=\frac{1}{6}$

 

 

Bài 102:(Ispectorgadget)

Cho $a,b,c$ thực dương. Chứng minh:
$$1<\frac{a}{a+b}+\frac{b}{b+c}+\frac{c}{c+a}+\sqrt{\frac{2abc}{(a+b)(b+c)(a+c)}} \le 2$$
Liệu có thể thay số 1 trong bất đẳng thức trên bởi một số lớn hơn được không?

 

 

Bài giải:

(WhjteShadow)$\bullet$ Vế trái:
Do $a,b,c$ là các số thực dương nên $a+b<a+b+c\\ b+c<a+b+c\\ c+a<a+b+c$
Từ đó ta có:
$$\frac{a}{a+b}+\frac{b}{b+c}+\frac{c}{c+a}>\frac{a+b+c}{a+b+c}=1$$
$$\sqrt{\frac{2abc}{(a+b)(b+c)(a+c)}}>0$$ 
Nên ta có:
$$1<\frac{a}{a+b}+\frac{b}{b+c}+\frac{c}{c+a}+\sqrt{\frac{2abc}{(a+b)(b+c)(a+c)}}$$
Ta không thể thay $1$ bằng số khác ( Thật vậy thử với $a,b\to 0$ )
$\bullet$ Vế phải:
Đặt $\frac{b}{a}=x,\frac{c}{b}=y,\frac{a}{c}=z$ ($x,y,z>0,xyz=1$) ta cần chứng minh:
$$\frac{1}{x+1}+\frac{1}{y+1}+\frac{1}{z+1}+\sqrt{\frac{2}{(x+1)(y+1)(z+1)}}\leq 2$$
Quy đồng và đưa bất đẳng thức về the0 ngôn ngữ $p,q,r$ với chú ý $r=1$ ta có bất đẳng thức tương đương:
$$\frac{3+2p+q}{2+p+q}+\sqrt{\frac{2}{2+p+q}}\leq 2$$
$$\Leftrightarrow \sqrt{\frac{2}{2+p+q}}\leq \frac{1+q}{2+p+q}$$
$$\Leftrightarrow 2(2+p+q)\leq (1+q)^2$$
$$\Leftrightarrow 3+2p\leq q^2$$
Điều này luôn đúng do $q^2\geq 3pr=3q\geq 2p+3$
Kết thúc chứng minh. Dấu đẳng thức xảy ra tại $a=b=c$ $\square$

 

 

Bài 103:

Cho $x,y,z >0$ thoả mãn: $\sum x\geq \sum \frac{1}{x}$.Chứng minh rằng: $$\sum x\geq \frac{3}{x+y+z}+\frac{2}{xyz}$$
___
NLT: Chú ý tiêu đề bài viết!

Bài giải:

$x+y+x \geq \frac{1}{x}+\frac{1}{y}+\frac{1}{z} \geq \frac{9}{x+y+z} $
$\Rightarrow x+y+z \geq 3 $
$\Rightarrow (x+y+z)^2 = \frac{(x+y+z)^2}{3}+\frac{2(x+y+z)^2}{3} \geq 3 + \frac{2(\frac{1}{x}+\frac{1}{y}+\frac{1}{z})^2}{3} \geq 3 + 2( \frac{1}{xy}+\frac{1}{yz}+\frac{1}{zx})$
đpcm

 

Bài 104:
cho a,b,c >0 thoả mãn:a+b+c=6.CMR:
$\sum \frac{a}{\sqrt{(b+2)(b^{2}-b+2)}}\geq \frac{3}{2}$

 

Bài giải:

 

sữ dụng đánh giá sau:
$\sqrt{(b+2)(b^{2}-b+2)}\leq \frac{b+2+b^{2}-b+2}{2}\leq \frac{b^{2}+4}{2}$
ta đưa bài toán về chứng minh
$\sum \frac{a}{b^{2}+4}\geq \frac{3}{4}$
ta có $\frac{a}{b^{2}+4}=\frac{a}{4}-\frac{ab^{2}}{4(b^{2}+4)}\geq \frac{a}{4}-\frac{ab^{2}}{4.4b}\geq \frac{a}{4}-\frac{ab}{16}$
làm tương tự 2 phânt thức kia kết hợp với bđt $ab+bc+ca\leq \frac{(a+b+c)^{2}}{3}\leq 12$ là ra

 

Bài 105:

Cho a,b thuộc [0;1]. CMR:
$a+b+\frac{(a-1)(b-1)}{ab+1}\geq 1+\frac{a^2b+ab^2}{2(ab+1)}$


(thpt YD2)

Bài giải:

 

 

Ta có bất đẳng thức tương đương với ^^~
$$a+b-\frac{ab(a+b)}{2(ab+1)}\geq 1-\frac{(a-1)(b-1)}{ab+1}$$
$$\Leftrightarrow (a+b).\left(1-\frac{ab}{2(ab+1)}\right)\geq \frac{a+b}{ab+1}$$
$$\Leftrightarrow \frac{ab+2}{2(ab+1)}\geq \frac{1}{ab+1}$$
$$\Leftrightarrow \frac{ab+2}{2}\geq 1$$
$$\Leftrightarrow ab\geq 0$$
Bất đẳng thức cuối luôn đúng nên ta có điều phải chứng minh.Đẳng thức xảy ra khi $a,b$ có 1 số bằng 0 ' '~

 

 

Bài 106:
các bạn mình là mem mới. xin các bạn giúp mình bài này với,minh rat can su tro giup cua cac ban;
chứng minh bất dẳng thúc này đúng với mọi tam giác ABC:
$2\sqrt{2}\left ( \sin \dfrac{A}{2}+\sin \dfrac{B}{2}+\sin \dfrac{C}{2} \right )> \cos \dfrac{A-B}{\sqrt{15}}+\cos \dfrac{B-C}{\sqrt{15}}+\cos \dfrac{C-A}{\sqrt{15}}$

Bài giải:

 

 

Ta có $cos\frac{C}{2}\left ( sin\frac{A}{2}+sin\frac{B}{2} \right )$;

$cos\frac{B}{2}\left ( sin\frac{C}{2}+sin\frac{A}{2} \right )$;
$cos\frac{A}{2}\left ( sin\frac{B}{2}+sin\frac{C}{2} \right )$.
Cộng vế với vế các bất đảng thức trên ta được
$sin\frac{A+B}{2}+sin\frac{B+C}{2}+sin\frac{C+A}{2}$$<2\left ( sin\frac{A}{2}+sin\frac{B}{2}+sin\frac{C}{2} \right )$.
hay $cos\frac{A}{2}+cos\frac{B}{2}+cos\frac{C}{2}<2\left ( sin\frac{A}{2}+sin\frac{B}{2}+sin\frac{C}{2} \right )$.
Đến đây chỉ cần chứng minh $\sqrt{2}\left ( cos\frac{A}{2}+cos\frac{B}{2}+cos\frac{C}{2} \right )>\sum cos\frac{A-B}{\sqrt{15}}$ là xong.
Ta có $cos\frac{A}{2}+cos\frac{B}{2}=2cos\frac{A+B}{4}cos\frac{A-B}{4}>2.cos\frac{\pi }{4}cos\frac{A-B}{4}=\sqrt{2}cos\frac{A-B}{4}$.
Mặt khác $\frac{\left | A-B \right |}{4}<\frac{\left | A-B \right |}{\sqrt{15}}<\frac{\pi }{2}$.
Suy ra $cos\frac{A}{2}+cos\frac{B}{2}>\sqrt{2}cos\frac{A-B}{\sqrt{15}}$. (*)
Cộng vế với vế các BĐT tương tự (*) ta thu được đpcm.$\square$
 

Bài 107

cho a,b,c >0 thoả mãn:a+b+c=3.CMR:
$\sum a^{2}+\frac{ab+bc+ac}{a^{2}b+b^{2}c+c^{2}a}\geq 4$

 

Bài giải:

 

BĐT tương đương với
$(a)^2-2\sum{ab}+\dfrac{3\sum{ab}}{(\sum{a})(\sum{a^2b})}\geq 9-2\sum{ab})+\dfrac{3\sum{ab}}{3\sum{a^2}}= 9-2q+\dfrac{q}{9-2q}$

Cần chứng minh $9 - 2q +\dfrac{q}{9-2q} \geq 4 \Leftrightarrow (q-3,75)(q-3)\geq 0$ (đúng vì $q \leq 3$)

 

 

Bài 108:
Cho $a,b,c$ là $3$ cạnh tam giác.CMR: \[\sum \frac{(b+c-a)^{4}}{a(a+b-c)}\geq ab+bc+ac\]

 

 

Bài giải:

đặt:
$b+c-a=x$
$c+a-b=y$
$a+b-c=z$
$\Rightarrow a=\frac{y+z}{2};b=\frac{z+x}{2};c=\frac{x+y}{2}$

BDT cần cm tuơng đuơng
$\sum \frac{x^{4}}{\frac{(y+z)z}{2}}\geq \sum \frac{(x+y)(y+z)}{4}$
ta có:
$\sum \frac{x^{4}}{\frac{(y+z)z}{2}}\geq \frac{(\sum x^{2}) ^{2}}{\frac{1}{2}\sum (z^{2}+yz)}$

$\sum xy\leq \sum x^{2}\Rightarrow VT\geq \sum x^{2}$
ta can cm
$\sum x^{2}\geq \sum \frac{1}{4}\sum (x+y)(y+z)$
hay $\sum x^{2}\geq \sum xy$ ( hien nhien)
dau "=" xay ra khi x=y=z hay a=b=c

 

 

Bài 109: Chứng minh với mọi a,b,c không âm,không có 2 số nào đồng thời bằng không 
$\frac{1}{a^2+bc}+\frac{1}{b^2+ca}+\frac{1}{c^2+ab}\geq \frac{3}{ab+bc+ca}$

Đây là 1 bài toán khá hay. Nó là hệ quả của 1 BDT của anh Phạm Kim Hùng
Giải như sau:
Bất đẳng thức đã cho tương đương với:
$\sum \frac{a(b+c)}{a^2+bc}+\sum \frac{bc}{a^2+bc}\geq 3$

+++)
Theo Cauchy-Schwarz thì $\sum \frac{bc}{a^2+bc}\geq\frac{(ab+ac+bc)^2}{abc(a+b+c)+\sum b^2c^2}\geq 1$
+++)
Ta chưng minh:
$\sum \frac{a(b+c)}{a^2+bc}\geq 2$ (*)
Xét khai triển sau:
$(a-b)^2(b-c)^2(c-a)^2\geq 0\Leftrightarrow \sum a^4(b^2+c^2)+2abc\sum a^2(b+c)\geq 2\sum a^3b^3+6a^2b^2c^2+2abc\sum a^3$
(*)$\Leftrightarrow \sum a(b+c)(c^2+ab)(b^2+ac)\geq 2(a^2+bc)(b^2+ac)(c^2+ab)\Leftrightarrow 2abc\sum a^2(b+c)+\sum a^4(b^2+c^2)+abc\sum a^2(b+c)\geq 4a^2b^2c^2+2\sum a^3b^3+2abc\sum a^3$
Kết hợp khai triển trên và $2a^2b^2c^2+abc\sum a^2(b+c)\geq 0$ ta có điều phải chứng minh
Đẳng thức xảy ra khi và chỉ khi a=b,c=0 hoặc các hoán vị
Chứng minh của ta được hoàn tất.

 

Bài 120.Chứng minh với mọi a,b,c không âm 
$\frac{1}{\frac{1}{a+1}+\frac{1}{b+1}+\frac{1}{c+1}}-\frac{1}{\frac{1}{a}+\frac{1}{b}+\frac{1}{c}}\geq \frac{1}{3}$

 

 

 

Giải như sau:
Nhân tung lên ta được đpcm tương đương với:
$\frac{(a+1)(b+1)(c+1)}{ab+ac+bc+2(a+b+c)+3}-\frac{abc}{ac+ac+bc}\geq \frac{1}{3}$
Đến đây đặt x=a+b+c,y=ab+ac+bc,z=abc ta có điều trên tương đương với:$3xy+2y^2\geq 6xz+9z$
Điều này đúng do $(ac+ab+bc)^2\geq 3abc(a+b+c);(a+b+c)(ab+ac+bc)\geq 9abc$( theo AM-GM)
Vậy ta có đpcm
Đẳng thức xảy ra khi và chỉ khi a=b=c

 

Bài 121:

Cho $\bigtriangleup ABC$$\bigtriangleup ABC$ và M nằm trong tam giác. BC=a,CA=b,Ab=c
Gọi khoảng cách từ M=>BC là x, M=>AC là y,M=>AB là z.
cmr $\sqrt{x}+\sqrt{y}+\sqrt{z}\leq \sqrt{\frac{a^{2}+b^{2}+c^{2}}{2R}}$ ( R là bán kính đường tròn ngoại tiếp) và $\sqrt{x}+\sqrt{y}+\sqrt{z}\leq \sqrt{\frac{(a^{2}+b^{2}+c^{2})2S}{abc}}$ (S là diện tích tam giác) có giống nhau không vâyhj(Nếu giống biến đổi kiểu gì vậy?

 

Bài giải:

 

ĐẲng thức quen thuộc $R=\frac{abc}{4S}$ mà bạn 
Ta có điều phải chứng minh tương đương:
$$\left(\sqrt{x}+\sqrt{y}+\sqrt{z}\right)^2\leq \frac{(a^{2}+b^{2}+c^{2})2S}{abc}$$
Sử dụng bất đẳng thức $Cauchy-Schwarz$ và để ý $ax+by+cz=2S$ ta có:
$$\left(\sqrt{x}+\sqrt{y}+\sqrt{z}\right)^2\leq (ax+by+cz).\left(\frac{1}{a}+\frac{1}{b}+\frac{1}{c}\right)$$
$$=2S.\frac{ab+bc+ca}{abc}\leq \frac{(a^{2}+b^{2}+c^{2})2S}{abc}$$
Kết thúc chứng minh.Đẳng thức xảy ra tại $a=b=c$,tam giác của ta là tam giác đều $\square$

 

 

Bài 122:
Cho $a,b> 0, a+b=\frac{1}{2}. CM: \frac{1}{a^2+b^2}+\frac{10}{\sqrt{a}}+\frac{10}{\sqrt{b}}\geq 48.$

Bài giải:

 

 

Bạn đừng vội quá. Có gợi ý ch0 bạn đây 
$\bullet$ Áp dụng liên tiếp bất đẳng thức $AM-GM$ ta có:
$$\frac{1}{a^2+b^2}+\frac{4}{\sqrt{a}}+\frac{4}{\sqrt{a}}+\frac{4}{\sqrt{b}}+\frac{4}{\sqrt{b}}\geq 5.\sqrt[5]{\frac{4^4}{(a^2+b^2)ab}}$$
$$=5.\sqrt[5]{\frac{2^{11}}{8(a^2+b^2)ab}}= 5.\sqrt[5]{\frac{2^{11}}{4(a^2+b^2)2ab}}$$
$$\geq 5.\sqrt[5]{\frac{2^{11}}{(a+b)^4}}=5.\sqrt[5]{2^{15}}=40$$
$\bullet$ Và mặt khác ta lại có:
$$\frac{2}{\sqrt{a}}+\frac{2}{\sqrt{b}}\geq \frac{4}{\sqrt[4]{ab}}\geq \frac{4\sqrt{2}}{\sqrt{a+b}}$$
$$=4\sqrt{2}.\sqrt{2}=8$$
Cộng 2 bất đẳng thức cùng chiều trên ta có điều phải chứng minh thôi :") $\square$

Bài 123:

cho a,b,c>0 thoả mãn:a+b+c=$\frac{1}{2}$

,Tìm GTLN:$\sum \sqrt{\frac{(a+b)(b+c)}{(a+b)(b+c)+a+c}}$

 

Bài giải:

 

Đặt $x=a+b;y=b+c;z=c+a$
Có:
$P=\sum \sqrt{\frac{xy}{xy+z}}=\sum \sqrt{\frac{xy}{(x+z)(z+y)}}\leq\frac{1}{2}\sum (\frac{x}{x+z}+\frac{y}{z+y})=\frac{3}{2}\blacksquare$
$"=" <=>x=y=z=\frac{1}{6}$

Bài 124:

 

$Cho a,b,c> 0, CM:\sqrt{\frac{ab+bc+ca}{a^2+b^2+c^2}}+\frac{a+b+c}{\sqrt[3]{abc}}\geq 4$

Bài giải:

 

 Áp dụng trực tiếp AM-GM thôi:
$\sqrt{\frac{ab+bc+ca}{a^2+b^2+c^2}}+\frac{a+b+c}{\sqrt[3]{abc}}\geq 4\sqrt[4]{\left ( \frac{a+b+c}{3\sqrt[3]{abc}} \right )^3.\sqrt{\frac{ab+bc+ca}{a^2+b^2+c^2}}}$
Nên ta cần chứng minh:
$\left ( \frac{a+b+c}{3\sqrt[3]{abc}} \right )^3.\sqrt{\frac{ab+bc+ca}{a^2+b^2+c^2}}\geq 1\Leftrightarrow \frac{(a+b+c)^6}{(3^6a^2b^2c^2}.\frac{ab+bc+ca}{a^2+b^2+c^2}\geq 1$
Ta có:
$\frac{(a+b+c)^6}{3^6a^2b^2c^2}.\frac{ab+bc+ca}{a^2+b^2+c^2}\geq \frac{(a+b+c)^6(ab+bc+ca)}{3^3(ab+bc+ca)^3(a^2+b^2+c^2)}$
$=\frac{\left ( \frac{a^2+b^2+c^2+ab+bc+ca+ab+bc+ca}{3} \right )^3}{(a^2+b^2+c^2)(ab+bc+ca)^2}\geq 1$

Bài 125:

 

Cho x,y thuộc [-1;1]. CMR:
$\sqrt{x^2+(y-1)^2}+\sqrt{x^2+(y+1)^2}+\sqrt{(x-1)^2+y^2}+\sqrt{(x+1)^2+y^2}\geq 4$

Bài giải:

Sr nhìn không kĩ, Min chỉ c/m được $\ge 2\sqrt{2}$...
----

Tổng quát $x,y\in \mathbb{R}$ thì BĐT vẫn đúng:
Dự đoán điểm rơi ở $x=y=0$ nên ta có các đánh giá sau:
$$\sqrt{x^2+(y-1)^2}\ge \sqrt{(y-1)^2}=|1-y|\\ \sqrt{x^2+(y+1)^2}\ge \sqrt{(y+1)^2}=|y+1|\\ \sqrt{(x-1)^2+y^2}\ge \sqrt{(x-1)^2}=|1-x|\\\sqrt{(x+1)^2+y^2}\ge \sqrt{(x+1)^2}=|x+1|$$
Từ các điều trên suy ra:
$$LHS\ge |1-y|+|y+1|+|1-x|+|x+1|\ge |1-y+y+1+1-x+x+1|=4=RHS$$
Dấu bằng xảy ra khi $x=y=0\ \square$
----
P/s: Cái giả thiết làm được gì cho đời nhỉ?

 

Bài 126:

 

Cho $a,b,c>0$. Chứng minh rằng: \[ \frac{1}{2a+b}+\frac{1}{2b+c}+\frac{1}{2c+a}\geq \frac{2}{\sqrt[3]{(a+b)(b+c)(c+a)}}\]

 

Bài giải:

Bài này không khó nhưng phải tinh ý chút  :)
Nhân thêm 2 vế với $a+b+c$ và biến đổi 1 chút ta sẽ có:
$$\sum_{cyc}\frac{c}{2a+b}+\frac{1}{2}\sum_{cyc}\frac{b}{2a+b}+\frac{3}{2} \ge \frac{2(a+b+c)}{\sqrt[3]{(a+b)(b+c)(c+a)}}$$
Theo C-S:
$$\sum_{cyc}\frac{c}{2a+b} \ge \frac{(a+b+c)^2}{3(ab+bc+ca)}$$
$$\sum_{cyc}\frac{b}{2a+b} \ge \frac{(a+b+c)^2}{a^2+b^2+c^2+2(ab+bc+ca)}=1$$
Vậy ta cần chứng minh:
$$\frac{(a+b+c)^2}{3(ab+bc+ca)}+2 \ge \frac{2(a+b+c)}{\sqrt[3]{(a+b)(b+c)(c+a)}}$$
Theo AM-GM:
$$\frac{(a+b+c)^2}{3(ab+bc+ca)}+2 \ge 3\sqrt[3]{\frac{(a+b+c)^2}{3(ab+bc+ca)}}$$
Bài toán sẽ được giải quyết nếu ta chứng minh :
$$3\sqrt[3]{\frac{(a+b+c)^2}{3(ab+bc+ca)}} \ge \frac{2(a+b+c)}{\sqrt[3]{(a+b)(b+c)(c+a)}}$$
Tương đương với:
$$(a+b)(b+c)(c+a) \ge \frac{8}{9}(ab+bc+ca)(a+b+c) \iff c(a-b)^2+a(b-c)^2+b(c-a)^2 ]ge 0$$
Luôn đúng.Đẳng thức xảy ra khi $a=b=c$.

Bài 127:

 

Chứng minh rằng: $\frac{3(a^4+b^4+c^4)}{(a^2+b^2+c^2)^2}+\frac{ab+bc+ca}{a^2+b^2+c^2}\geq 2.$

 

Bài giải:

$3\frac{\sum x^{4}}{(\sum x^{2})}+\frac{\sum xy}{\sum x^{2}}\geq 2 \Leftrightarrow 3(\sum x^{4})+\sum xy\sum x^{2}\geq 2((\sum x^{2})^{2})$

hay

$x^{4}+y^{4}+z^{4}+xy(x^{2}+y^{2})+yz(y^{2}+z^{2})+zx(z^{2}+x^{2})\geq 4(x^{2}y^{2}+y^{2}z^{2}+z^{2}x^{2} )-xyz(x+y+z)$

$xyz(x+y+z)\leq x^{2}y^{2}+y^{2}z^{2}+z^{2}x^{2}\Rightarrow -xyz(x+y+z)\geq -(x^{2}y^{2}+y^{2}z^{2}+z^{2}x^{2})$


nen ta can chung minh

$x^{4}+y^{4}+z^{4}+xy(x^{2}+y^{2})+yz(y^{2}+z^{2})+zx(z^{2}+x^{2})\geq 3(x^{2}y^{2}+y^{2}z^{2}+z^{2}x^{2} )$

ap dung AM-GM ta có

$x^{4}+y^{4}+z^{4}\geq x^{2}y^{2}+y^{2}z^{2}+z^{2}x^{2}$



$xy(x^{2}+y^{2})+yz(y^{2}+z^{2})+zx(z^{2}+x^{2})\geq 2(x^{2}y^{2}+y^{2}z^{2}+z^{2}x^{2})$

cong vo theo ve 2 bdt tren ta co dpcm
dau "=" khi $x=y=z$

 

Bài 128:

Với mọi a,b,c>0chứng minh rằng:
$\frac{(a+b+c)^3}{a^2b+b^2c+c^2a+3abc}\geq \frac{9}{2}$

 

 

Bài giải:

$a^2b+b^2c+c^2a+abc\leq \frac{4}{27}(a+b+c)^3$ 

 

Không mất tính tổng quát ta giả sử b nằm giữa c và a.
suy ra $(b-a)(b-c)\leq 0\rightarrow c(b-a)(b-c)\leq 0$
suy ra $b^2c+c^2a\leq bc^2+abc$
$\Leftrightarrow b^2c+c^2a+a^2b+abc\leq bc^2+a^2b+2abc\leq \frac{1}{2}2b(a+c)^2$
$\leq \frac{1}{2}\left [ \frac{(2b+a+c+a+c)}{3} \right ]^3$$=\frac{1}{2}\frac{\left [ 2(a+b+c) \right ]^3}{27}=4$
dấu đẳng thức xảy ra khi và chỉ khi a=b=c=1 hoặc a=2,b=1,c=0.

 

và $2abc\leq \frac{2}{27}(a+b+c)^3$
nên $\frac{(a+b+c)^3}{a^2b+b^2c+c^2a+3abc}\geq \frac{1}{\frac{4}{27}+\frac{2}{27}}=\frac{9}{2}$

 

 

 

 

 

 


Bài viết đã được chỉnh sửa nội dung bởi phanquockhanh: 23-07-2013 - 21:14


#5
CD13

CD13

    Thượng úy

  • Thành viên
  • 1456 Bài viết

Bài 41:

Cho a,b,c là các số dương, chứng minh rằng

$$\sqrt{\frac{2a}{a+b}}+\sqrt{\frac{2b}{b+c}}+\sqrt{\frac{2c}{c+a}}\leq\frac{3}{\sqrt{2}}$$

Giải:

Áp dụng BCS ta có:

$\left ( \sum \sqrt{\frac{a}{a+b}} \right )^{2}\leq (\sum (a+b))(\sum \frac{b}{(b+c)(a+b)})= \frac{4(a+b+c)(ab+bc+ca)}{\prod (a+b)}$

Dế thấy ta chỉ cần CM bđt phụ sau

$8(a+b+c)(ab+bc+ca)\leq 9\prod (a+b)$ (quá quen thuộc)

Đẳng thức xảy ra khi và chỉ khi a=b=c

Cách khác:

Đặt $x=\sqrt{\dfrac{b}{a}}, y=\sqrt{\dfrac{c}{b}}, z=\sqrt{\dfrac{a}{c}}$, $xyz=1$ ta cần chưng minh:

$$\dfrac{1}{\sqrt{1+x^2}}+\dfrac{1}{\sqrt{1+y^2}}+\dfrac{1}{\sqrt{1+z^2}} \leq \dfrac{3}{\sqrt{2}}$$

Giả sử $x=max${$x,y,z$} thì $yz \leq 1$

Ta có $\dfrac{1}{2} \left( \dfrac{1}{\sqrt{1+y^2}}+\dfrac{1}{\sqrt{1+z^2}} \right)^2 \leq \dfrac{1}{1+y^2}+\dfrac{1}{1+z^2} \leq \dfrac{2}{1+yz}$

$\Rightarrow \dfrac{1}{\sqrt{1+y^2}}+\dfrac{1}{\sqrt{1+z^2}} \leq \dfrac{2}{\sqrt{1+yz}}$

Lại có $\dfrac{1}{\sqrt{1+x^2}} \leq \dfrac{\sqrt{2}}{x+1}$

Vậy ta chỉ cần chứng minh:

$$\dfrac{2}{\sqrt{1+yz}}+\dfrac{\sqrt{2}}{x+1} \leq \dfrac{3}{\sqrt{2}}$$

Ta có: $\dfrac{2}{\sqrt{1+yz}}+\dfrac{\sqrt{2}}{x+1} -\dfrac{3}{\sqrt{2}}=\dfrac{-(\sqrt{x+1}-\sqrt{2x})^2}{2(x+1)} \leq 0$

Bài toán được chứng minh.

 

 

 

Bài 42:

Cho các  số thực  $x;y;z >0$ ;$x^2 +y^2 +z^2 =\frac{1}{3}$.Tìm giá trị nhỏ nhất của biểu thức :

$P= \frac{xy +yz +zx}{x+y+z} +\frac{2(x+y+z)}{9(xy+yz+zx)}$

Giải:

$P= \frac{xy +yz +zx}{x+y+z} +\frac{2(x+y+z)}{9(xy+yz+zx)}$

   

$\geq \frac{(x+y+z)^{2}-(x^{2}+y^{2}+z^{2})}{2(x+y+z)}+\frac{2(x+y+z)}{3(x+y+z)^{2}}$

   

$=\frac{(x+y+z)^{2}-\frac{1}{3}}{2(x+y+z)}+\frac{2}{3(x+y+z)}$

   

$=\frac{x+y+z}{2}+\frac{5}{6(x+y+z)}$

 

Đặt $x+y+z=t$, ta có: $(x+y+z)^{2}\leq 3(x^{2}+y^{2}+z^{2})=3.\frac{1}{3}=1\Rightarrow 0< t\leq 1$

 

Khi đó: $P\geq \frac{t}{2}+\frac{5}{6t}$

 

Xét hàm: $f(t)= \frac{t}{2}+\frac{5}{6t}$ trên (0;1]

 

$f'(t)= \frac{3t^{2}-5}{6t^{2}}=0\Leftrightarrow t=\pm \sqrt{\frac{5}{3}}$

 

Dựa vào bảng biến thiên, hàm $f(t)$ nghịch biến trên (0;1]

 

$f(t)\geq f(1)=\frac{4}{3}$

 

Vậy minP=$\frac{4}{3}$, đạt được khi $x=y=z=\frac{1}{3}$

 

 

 

Bài 43:

Chứng minh :
$\frac{x^{3}}{y^{3}+8}+\frac{y^{3}}{z^{3}+8}+\frac{z^{3}}{x^{3}+8}\geq \frac{1}{9}+\frac{2(xy+yz+zx)}{27}$
với $x,y,z>0$ và $x+y+z=3$.

Giải:

Áp dụng bất đẳng thức Cauchy:

 

$\frac{x^{3}}{(y+2)(y^{2}-2y+4)}+\frac{y+2}{27}+\frac{y^{2}-2y+4}{27}\geq \frac{x}{3}$

 

Tương tự ta có: $\frac{y^{3}}{(z+2)(z^{2}-2z+4)}+\frac{z+2}{27}+\frac{z^{2}-2z+4}{27}\geq \frac{y}{3}$

 

                         $\frac{z^{3}}{(x+2)(x^{2}-2x+4)}+\frac{x+2}{27}+\frac{x^{2}-2x+4}{27}\geq \frac{z}{3}$

 

Đặt vế trái của bất đẳng thức cần chứng minh là $A$

 

Cộng vế với vế ba bất đẳng thức cùng chiều trên ta có:

 

$A+\frac{(x^{2}+y^{2}+z^{2}-(x+y+z)+18)}{27}\geq 1$

 

hay $A+\frac{(x+y+z)^2}{27}\geq \frac{4}{9}+\frac{2(xy+yz+zx)}{27}$

 

hay $A\geq \frac{1}{9}+\frac{2(xy+yz+zx)}{27}$

 

 

 

Bài 44:

Cho $x,y,z \geq 0$ và $x+y+z=1$.

Tìm GTLN của $P=x(y-z)^4+y(z-x)^4+z(x-y)^4$

Giải:

Do vai trò bình đẳng của $x,y,z$ trong $P$ nên ta có thể giả sử: $0\leq z\leq y\leq x$

 

Khi đó ta có:  $P=x(y-z)^{4}+y(x-z)^{4}+z(x-y)^{4}\leq x(y+z)^{4}+yx^{4}+zx^{4}$

 

Đặt $d=y+z$ thì $P\leq xd^{4}+dx^{4}=xd(x^{3}+d^{3})=xd[(d+x)^{3}-3xd(d+x)]=xd(1-3xd)$     (vì $x+d=1$)

 

Khi đó: $P\leq -3\left ( xd-\frac{1}{6} \right )^{2}+\frac{1}{12}\leq \frac{1}{12}$

 

$P=\frac{1}{12}$ khi và chỉ khi $x=\frac{3+\sqrt{6}}{6},y=\frac{3-\sqrt{6}}{6},z=0$

 

Vậy giá trị lớn nhất của $P$ là $\frac{1}{12}$ đạt được khi và chỉ khi $(x,y,z)$ là một hoán vị của $\left (\frac{3+\sqrt{6}}{6},\frac{3-\sqrt{6}}{6},0 \right )$

 

 

 

Bài 45:

CMR nếu x,y>0 và $x+y\leq 1$ thì $\frac{1}{x^2+y^2}+\frac{2}{xy}+4xy\geq 11$

Giải:

Đặt vế trái của biểu thức cần chứng minh là $A$.

 

Ta có: $A=\left ( \frac{1}{x^{2}+y^{2}}+\frac{1}{2xy} \right )+\left (4xy+\frac{1}{4xy} \right )+\frac{5}{4xy}$

 

Áp dụng bất đẳng thức $\frac{1}{a}+\frac{1}{b}\geq \frac{4}{a+b}$ ta có:

 

                           $\frac{1}{x^{2}+y^{2}}+\frac{1}{2xy}\geq \frac{4}{(x+y)^{2}}\geq 4$

 

Áp dụng bất đẳng thức Cauchy ta có: $4xy+\frac{1}{4xy}\geq 2$

 

                                                           $\frac{5}{4xy}\geq \frac{5}{(x+y)^{2}}=5$

 

Cộng vế với vế ba bất đẳng thức trên ta có đpcm

 

 

Bài 46:

Cho x,y,z >0.Chứng minh rằng :

$\frac{\sqrt{x^2+2y^2}}{z}+\frac{\sqrt{y^2+2z^2}}{x}+\frac{\sqrt{z^2+2x^2}}{y} \geq 3\sqrt{3}$

Giải:

Ta dễ cm được bđt sau: $\sqrt{a^{2}+b^2+c^2}\geq \frac{a+b+c}{\sqrt{3}}$
 

Áp dụng bđt trên, ta có $(VT)\geq \sum \frac{x+2y}{\sqrt{3}z} $ $\geq \frac{1}{\sqrt{3}}(\sum \frac{3\sqrt[3]{xy^{2}}}{z}) \geq \sqrt{3}.3\sqrt[3]{\prod \frac{\sqrt{xy^{2}}}{z}}=3\sqrt{3}$

 

Dấu = đạt được khi x=y=z.

Cách khác:

Áp dụng BĐT Cauchy-Schwarz thì $(x^2+2y^2)(1+ 2) \ge (x+2y)^2$.

Do đó $\sum \frac{ \sqrt{x^2+2y^2}}{z} \ge \frac{1}{ \sqrt 3} \cdot \left( \frac{$x+2y}{z}+ \frac{y+2z}{x}+ \frac{z+2x}{y} \right)$

Lại có $\sum \frac{x+2y}{z} = \sum \frac xz + \sum \frac{2y}{z} \ge 9$.

Vậy $\sum \frac{ \sqrt{x^2+2y^2}}{z} \ge 3 \sqrt 3$.

Dấu đẳng thức xảy ra khi và chỉ khi $x=y=z$.

 

 

 

Bài 47:

Cho $a,b,c>0$. Chứng minh rằng

                $\frac{ab}{a^2+ab+b^2}+\frac{bc}{b^2+bc+c^2}+\frac{ca}{c^2+ca+a^2}\leqslant \frac{a}{2a+b}+\frac{b}{2b+c}+\frac{c}{2c+a}$

Giải:

Ta có:$\frac{a}{2a+b}+\frac{b-a}{a+2b} \geqslant \frac{ab}{a^2+ab+b^2}$

          $\Leftrightarrow (a-b)^2.\frac{a^2+4ab+b^2}{3(2a+b)(a+2b)(a^2+ab+b^2)}\geqslant 0$

Làm tương tự ta có:$\frac{ab}{a^2+ab+b^2}+\frac{bc}{b^2+bc+c^2}+\frac{ca}{c^2+ca+a^2}\leqslant \frac{a}{2a+b}+\frac{b}{2b+c}+\frac{c}{2c+a}+\sum \frac{b-a}{a+2b}$

Mà ta có:$\sum \frac{b-a}{a+2b}=\frac{3}{2}-\sum \frac{3a}{2(a+2b)}\leqslant \frac{3}{2}-\frac{3(a+b+c)^2}{2(a+b+c)^2}=0$

Vậy   $\frac{ab}{a^2+ab+b^2}+\frac{bc}{b^2+bc+c^2}+\frac{ca}{c^2+ca+a^2}\leqslant \frac{a}{2a+b}+\frac{b}{2b+c}+\frac{c}{2c+a}$

 

 

 

Bài 48:

Chứng minh rằng với mọi số dương $a, b, c, d$, ta có :
$$\dfrac{a}{3a^2 + 2b^2 + c^2} + \dfrac{b}{3b^2 + 2c^2 + d^2} + \dfrac{c}{3c^2 + 2d^2 + a^2} + \dfrac{d}{3d^2 + 2a^2 + b^2} \le \dfrac{1}{6}\left (\dfrac{1}{a} + \dfrac{1}{b} + \dfrac{1}{c} + \dfrac{1}{d} \right )$$

Giải:

Ta có: $3a^{2}+2b^{2}+c^{2}=2(a^{2}+b^{2})+(a^{2}+c^{2})\geq 4ab+2ac$

 

Do đó: $\frac{a}{3a^{2}+2b^{2}+c^{2}}\leq \frac{1}{4b+2c}\leq \frac{1}{9}\left ( \frac{1}{b}+\frac{1}{2c} \right )$

 

Tương tự:  $\frac{b}{3b^{2}+2c^{2}+d^{2}}\leq \frac{1}{9}\left ( \frac{1}{c}+\frac{1}{2d} \right )$

 

                   $\frac{c}{3c^{2}+2d^{2}+a^{2}}\leq \frac{1}{9}\left ( \frac{1}{d}+\frac{1}{2a} \right )$

 

                   $\frac{d}{3d^{2}+2a^{2}+b^{2}}\leq \frac{1}{9}\left ( \frac{1}{a}+\frac{1}{2b} \right )$

 

Cộng vế với vế bốn bất đẳng thức cùng chiều trên ta có đpcm

 

 

 

Bài 49:

Cho $x,y,z>0$ và $xyz \leqslant 1$

Chứng minh rằng $\frac{x}{y^2}+\frac{y}{z^2}+\frac{z}{x^2}+\frac{3}{x+y+z}\geqslant 4$

Giải:

Đặt $x=\frac{1}{a},y=\frac{1}{b},z=\frac{1}{c}$ thì điều kiện đã cho trở thành $abc\geq 1$ và bất đẳng thức cần chứng minh được viết lại thành:

 

                $\frac{b^{2}}{a}+\frac{c^{2}}{b}+\frac{a^{2}}{c}+\frac{3}{ab+bc+ca}\geq 4$

 

Áp dụng bất đẳng thức Cauchy ta có: $\frac{b^{2}}{a}+\frac{c^{2}}{b}+\frac{a^{2}}{c}\geq a+b+c$

 

                                                       và $ab+bc+ca\leq \frac{1}{3}(a+b+c)^{2}$

 

Đặt vế trái của biểu thức cần chứng minh là $A$.

 

Ta có: 

 

$A\geq a+b+c+\frac{9}{(a+b+c)^{2}}=\left (\frac{a+b+c}{3}+\frac{a+b+c}{3}+\frac{9}{(a+b+c)^{2}} \right )+\frac{a+b+c}{3}\geq 4$

Cách khác:

$\sum \frac{a}{b^{2}}\geq \frac{1}{a}\Leftrightarrow \sum a^{3}c^{2}\geq abc(ab+bc+ca)$ đúng

$\sum \frac{1}{a}+\frac{3}{a+b+c}= 3.\frac{ab+bc+ca}{abc}+\frac{3}{a+b+c}\geq 4\sqrt[4]{\frac{(a+b+c)^{3}}{9a^{3}b^{3}c^{3}(a+b+c)}}\geq 4\sqrt[4]{\frac{(ab+bc+ca)^{2}}{3abc(a+b+c)}}\geq 4$ đ.p.c.m

 

 

 

Bài 50:

Cho $a,b,c>0$. Chứng minh rằng 

                      $\frac{a^2}{a^2+ab+b^2}+\frac{b^2}{b^2+bc+c^2}+\frac{c^2}{c^2+ca+a^2}\geqslant 1$

Giải:

BĐT cần cm: $\sum \frac{a^{2}}{a^{2}+ab+b^{2}}\geq 1$

 

         $\Leftrightarrow \sum \frac{(\frac{a}{b})^{2}}{(\frac{a}{b})^{2}+\frac{a}{b}+1}\geq 1$

 

       Đặt $\frac{a}{b}=x,\frac{b}{c}=y,\frac{c}{a}=z$, thì xyz=1

 

Ta cần CM: $\sum \frac{x^{2}}{x^{2}+x+1}\geq 1$

 

ĐẶt $x=\frac{1}{m}$, $y=\frac{1}{n},z=\frac{1}{p}$, thì  mnp=1, ta cần Cm: 

$\sum \frac{1}{m^{2}+m+1}\geq 1$

 

 

 

Bài 51:

Cho a,b,c là các số dương  CMR :$\frac{1}{a+b}+\frac{1}{b+c}+\frac{1}{a+c}+\frac{1}{2\sqrt[3]{abc}}\geq \frac{(a+b+c+\sqrt[3]{abc})^{2}}{(a+b)(b+c)(a+c)}$

 

Giải:

Theo BĐT Schawrz :

$VP=\frac{c^{2}}{c^{2}(a+b)}+\frac{a^{2}}{a^{2}(b+c)}+\frac{b^{2}}{b^{2}(a+c)}+\frac{\sqrt[3]{(abc)^{2}}}{2abc}\geq \frac{(a+b+c+\sqrt[3]{abc})^{2}}{c^{2}(a+b)+a^{2}(b+c)+b^{2}(a+c)+2abc}=\frac{(a+b+c+\sqrt[3]{abc})^{2}}{(a+b)(b+c)(c+a)}$

Đây là đpcm. Đẳng thức xảy ra khi $a=b=c$

 

 

 

Bài 52:

Cho các số thực duơng $a,b,c$ thoả mãn: $a+b+c=3$. Chứng minh rằng:

 

$\sqrt{a}+\sqrt{b}+\sqrt{c}\geq \left ( a^{2}+b^{2}+c^{2} \right )\left ( 4- \frac{1}{a^{2}}-\frac{1}{b^{2}}-\frac{1}{c^{2}}\right )$

Giải:

Ta có bđt quen thuộc

$a+b+c=3\Rightarrow \sum \sqrt{a}\geq \sum bc$,thay $3=\frac{27}{(a+b+c)^{2}}$

Bây giờ ta sẽ chứng minh 

$ab+bc+ca-(a^{2}+b^{2}+c^{2})(4-\frac{1}{a^{2}}-\frac{1}{b^{2}}-\frac{1}{c^{2}})\geq 0\Leftrightarrow (a-b)^{2}\left [ \frac{2(ac+bc)^{2}+2(ab+bc+ca)c^{2}+4abc^{2}}{9a^{2}b^{2}c^{2}} -1\right ]+(a-c)(b-c)\left [ \frac{(a^{2}+b^{2})(a+c)(b+c)+2ab(ab+bc+ca)+2abc(a+b)}{9a^{2}b^{2}c^{2}}-1 \right ]= M(a-b)^{2}+N(a-c)(b-c)\geq 0$

Giả sử c là max ,sẽ thấy ngay ab khônng lớn hơn 1

 $M\geq 0\Leftrightarrow 2(a+b)^{2}+2(ab+bc+ca)+4ab> 10ab>9a^{2}b^{2}$ đúng theo giả thiết

$N\geq 2ab(a+c)(b+c)+2(ab+bc+ca)ab+2abc(a+b)\geq 9a^{2}b^{2}c^{2}\Leftrightarrow 4(ab+bc+ca)+2c(a+b)+2c^{2}\geq 7abc^{2}$

vì $2abc^{2}\geq 2a^{2}b^{2}c^{2}$

Nếu a+b $\geq \frac{1}{4}$,chia cả 2 vế cho c,ta được $4(\frac{ab}{c}+a+b)+6\geq 4.\frac{1}{4}+6= 7\geq 7abc$

Nếu a+b ngược lại,hay $a+b\leq \frac{1}{4}\Rightarrow c\geq \frac{11}{4}$

ta được $4(ab+bc+ca)+2c(a+b)+2c^{2}\geq \frac{11}{2}(a+b+c)= \frac{33}{2}= 16,5$

$7abc^{2}\leq \frac{7}{64}.1< 1< 16,5$

Vậy cả 2 trường hợp của a+b thì N đều dương

Vậy ta có M,N dương,mà giả sử c là max thì (a-c)(b-c) không âm

Vậy bài toán được chứng minh

Đẳng thức xảy ra khi và chỉ khi a=b=c=1

 

 

 

Bài 53:

Cho a,b,c là các số thực dương. Chứng minh rằng:

$\frac{a^{4}}{1+a^{2}b}+\frac{b^{4}}{1+b^{2}c}+\frac{c^{4}}{1+c^{2}a}\geqslant \frac{abc(a+b+c)}{1+abc}$

Giải:

Áp dụng bđt CS ta có vế trái $\geq \frac{\left ((\sum a^{2})^{2} \right )}{3+\sum a^{2}b}$

Ta phải cm $\frac{\left ((\sum a^{2})^{2} \right )}{3+\sum a^{2}b}\geq \frac{abc(a+b+c)}{1+abc}$

Biến đổi tương đương ta phải CM

$(\sum a^{2})^{2}(1+abc)\geq abc(a+b+c)(3+\sum a^{2}b)\Leftrightarrow (\sum a^{2})^{2}+abc(\sum a^{2})^{2}\geq 3abc(a+b+c)+abc(a+b+c)(\sum a^{2}b)$

Ta dễ dàng CM đc $(\sum a^{2})^{2} \geq3abc(a+b+c)$

Mặt khác ta lại có $abc(\sum a^{2})^{2} \geq abc(a+b+c)(\sum a^{2}b)\Leftrightarrow (\sum a^{2})^{2} \geq (a+b+c)(\sum a^{2}b)\Leftrightarrow \sum a^{4}+\sum2a^{2}b^{2} \geq \sum a^{3}b+\sum ab^{2}c+\sum a^{2}c^{2}$

Đến đây ta thấy bđt hiển nhiên đúng vì ta có thể dễ dàng cm theo bđt AM-GM các bđt $\sum a^{4} \geq a^{3}b$ và $\sum a^{2}b^{2} \geq \sum ab^{2}c$

Do đó ta có đpcm

Cách khác:

Chia cả 2 vế cho $abc$ ta cần chứng minh 

        $\frac{\frac{a^3}{b}}{c+a^2bc}+\frac{\frac{b^3}{c}}{a+ab^2c}+\frac{\frac{c^3}{a}}{b+abc^2}\geqslant \frac{a+b+c}{1+abc}$ (*)

$\Leftrightarrow \frac{\frac{a^4}{ab}}{c+a^2bc}+\frac{\frac{b^4}{bc}}{a+ab^2c}+\frac{\frac{c^4}{ca}}{b+abc^2}\geqslant \frac{a+b+c}{1+abc}$

Áp dụng B.C.S ta có $\frac{\frac{a^4}{ab}}{c+a^2bc}+\frac{\frac{b^4}{bc}}{a+ab^2c}+\frac{\frac{c^4}{ca}}{b+abc^2}\geqslant \frac{(\frac{a^2}{\sqrt{ab}}+\frac{b^2}{\sqrt{bc}}+\frac{c^2}{\sqrt{ca}})^2}{(a+b+c)(1+abc)}$

Do vậy ta chỉ cần chứng minh $\frac{(\frac{a^2}{\sqrt{ab}}+\frac{b^2}{\sqrt{bc}}+\frac{c^2}{\sqrt{ca}})^2}{(a+b+c)(1+abc)}\geqslant \frac{a+b+c}{1+abc}$

                          $\Leftrightarrow \frac{a^2}{\sqrt{ab}}+\frac{b^2}{\sqrt{bc}}+\frac{c^2}{\sqrt{ca}}\geqslant a+b+c$

Nhưng rõ ràng bất đẳng thức trên luôn đúng theo B.C.S và AM-GM

                $\frac{a^2}{\sqrt{ab}}+\frac{b^2}{\sqrt{bc}}+\frac{c^2}{\sqrt{ca}}\geqslant \frac{(a+b+c)^2}{\sqrt{ab}+\sqrt{bc}+\sqrt{ca}}\geqslant \frac{(a+b+c)^2}{a+b+c}=a+b+c$

Vậy ta có đpcm

Đẳng thức xảy ra khi $a=b=c>0$

 

 

 

Bài 54:

Cho các số dương $a,b,c$ thoả mãn $abc \leq 1$. Chứng minh rằng:

 

        $\frac{1}{a^{5}(b+2c)^{2}}+\frac{1}{b^{5}(c+2a)^{2}}+\frac{1}{c^{5}(a+2b)^{2}}\geq \frac{1}{3}$

Giải:

Đặt $ (\frac{1}{x},\frac{1}{y},\frac{1}{z})=(a,b,c)$, suy ra $xyz \geq 1$

Ta được:

$VT=\sum \dfrac{x^5y^2z^2}{(2y+z)^2} \geq \sum \dfrac{x^3}{(2y+z)^2}$

Áp dụng AM-GM ta có:

$\dfrac{x^3}{(2y+z)^2}+ \dfrac{2y+z}{27} + \dfrac{2y+z}{27} \geq \dfrac{x}{3}$

Tương tự và cộng lại ta được:

$VT \geq \dfrac{x+y+z}{9} \geq \dfrac{\sqrt[3]{xyz}}{3} \geq \frac{1}{3}$ 

dấu = khi $ a=b=c=1$

 

 

 

Bài 55:

Cho các số thực dương x, y thoả mãn $(x + y - 1)^2 = xy$. Tìm giá trị min của biểu thức:
$P = \frac{1}{xy} + \frac{1}{x^2+  y^2} + \frac{\sqrt[]{xy}}{x + y}$

 

Giải:

Giả thiết: $(x + y - 1)^2 = xy$$\Leftrightarrow \begin{bmatrix} x+y=\sqrt{xy}+1 & \\ x+y=-\sqrt{xy}+1 & \end{bmatrix}$
 
$P = \frac{1}{xy} + \frac{1}{x^2+  y^2} + \frac{\sqrt[]{xy}}{x + y}$
 

      =$\frac{1}{xy}+\frac{1}{(x+y)^{2}-2xy}+\frac{\sqrt{xy}}{x+y}$

    

    Ở đây sẽ có 2 TH, nhưng ta nhận ra rằng P có giá trị nhỏ nhất trong TH: $x+y=\sqrt{xy}+1$

 

Vì: $\frac{1}{xy}+\frac{1}{-xy+2\sqrt{xy}+1}+\frac{\sqrt{xy}}{1+\sqrt{xy}}< \frac{1}{xy}+\frac{1}{-xy-2\sqrt{xy}+1}+\frac{\sqrt{xy}}{1-\sqrt{xy}},(\forall x,y> 0)$

Do đó 

     P=$\frac{1}{xy}+\frac{1}{(\sqrt{xy}+1)^{2}-2xy}+\frac{\sqrt{xy}}{\sqrt{xy}+1}$

 

     =$\frac{1}{xy}+\frac{1}{-xy+2\sqrt{xy}+1}+\frac{\sqrt{xy}}{\sqrt{xy}+1}$

 

Đặt $\sqrt{xy}=t$ theo giả thiết, ta có: $xy=(x+y-1)^{2}\geq (2\sqrt{xy})\Leftrightarrow 3xy-4\sqrt{xy}+1=0\Leftrightarrow \frac{1}{3}\leq \sqrt{xy}\leq 1$ hay $\frac{1}{3}\leq t\leq 1$

Khi đó:

$P=\frac{1}{t^{2}}+\frac{1}{(-t^{2}+2t+1)}+\frac{t}{t+1}$

 

Xét hàm: $f(t)=\frac{1}{t^{2}}+\frac{1}{(-t^{2}+2t+1)}+\frac{t}{t+1}$ trên $[\frac{1}{3};1]$

 

Ta có: $f'(t)=\frac{-2}{t^{3}}-\frac{2(1-t)}{(-t^{2}+2t+1)^{2}}+\frac{1}{(t+1)^{2}}$

 

Ta thấy: $t^{3}\leq 1;(t+1)^{2}\geq 1\Rightarrow \frac{-2}{t^{3}}+\frac{1}{(t+1)^{2}}<0 ,\forall t\in[\frac{1}{3};1]$

Suy ra: $f'(t)< 0,\forall t\in [\frac{1}{3};1]$

 

$\Rightarrow$Hàm $f(t)$ nghịch biến trên $[\frac{1}{3};1]$

Suy ra $f(t)\geq f(1)=2$

 

Vậy $minP=2$, đạt được khi $x=y=1$

 

 

 

Bài 56:

Cho $x,\,y,\,z\in\left[1;\,3\right].$ Tìm giá trị nhỏ nhất của: $$P=\dfrac{36x}{yz}+\dfrac{2y}{xz}+\dfrac{z}{xy}$$

Giải:

Áp dụng BĐT AM-GM ta có:

 

$\dfrac{18x}{yz}+\dfrac{2y}{xz} \geq \dfrac{12}{z} \geq 4$   (1)

 

$\dfrac{9x}{yz} + \dfrac{z}{xy} \geq \dfrac{6}{y} \geq 2$      (2)

 

Mà: $\dfrac{9x}{yz} \geq 1$                         (3)

Lấy $(1)+(2)+(3) $theo vế ta được:

$P \geq 7$

Vậy $MinP=7$ khi $x=1,y=z=3$

 

 

 

Bài 57:

 Tìm giá trị nhỏ nhất của biểu thức:
$P=2\left(\dfrac{x^5}{y}+\dfrac{y^5}{x}\right)+x^8+y^8-4(1+xy)^2$
Với $x>0$ và $y>0$.
Giải:
$P=2\left(\dfrac{x^5}{y}+\dfrac{y^5}{x}\right)+x^8+y^8-4(1+xy)^2\geqslant 2.2\sqrt{\frac{x^5.y^5}{xy}}+2x^4.y^4-4-8xy-4x^2y^2=2(x^4y^4-2-4xy)=2(x^4y^4+3-4xy)-10\geqslant -10$

 

 

 

Bài 58:

Cho $a,b,c$ là các số thực thỏa mãn $a^2+b^2+c^2 \leqslant 8$

Tìm GTNN của $P=ab+bc+2ac$

Giải:

$2P+16 \geq (a+b+c)^2+b^2+(a+c)^2 \geq 0$

$\Leftrightarrow  P \geq -8$

$MinP=-8$ khi $(a,b,c)=(-2,0,2)$ hoặc $(2,0,-2)$

 

 

 

Bài 59:

Cho $x,y,z\in (0;1]$

 

$\sum \frac{1}{xy+1} \leq \frac{5}{x+y+z}$

 

Giải:

BĐT đã cho tương đương với $\sum \frac{x+y+z}{xy+1}\leqslant 5$

                $\Leftrightarrow \sum \frac{x+y}{xy+1}+\sum \frac{z}{xy+1}\leqslant 5$

Dễ thấy $x,y \in \left(0;1 \right ]\Rightarrow (1-x)(1-y)\geqslant 0\Rightarrow 1+xy\geqslant x+y$

           $\Rightarrow \frac{x+y}{xy+1}\leqslant 1\Rightarrow \sum \frac{x+y}{xy+1}\leqslant 3$    (*)

Do đó ta chỉ  cần chứng minh $\sum \frac{z}{xy+1}=\frac{z}{xy+1}+\frac{y}{xz+1}+\frac{x}{yz+1}\leqslant 2$

Ta sẽ chứng minh $\frac{z}{xy+1}\leqslant \frac{2z}{x+y+z}\Leftrightarrow x+y+z\leqslant 2xy+2$

              $\Leftrightarrow (1-x)(1-y)+(1-z)+xy\geqslant 0$, luôn đúng do giả thiết

Tương tự 2 bất đẳng thức còn lại ta có $\sum \frac{z}{xy+1}\leqslant \sum \frac{2z}{x+y+z}=2$     (**)

Từ (*) và (**) ta có ngay đcpm

Đẳng thức không xảy ra

 

 

 

Bài 60:

Cho $a,b,c$ không âm thỏa mãn $a^2+b^2+c^2=3$

Tìm GTNN của $P=\frac{a}{\sqrt{a^2+b+c}}+\frac{b}{\sqrt{b^2+c+a}}+\frac{c}{\sqrt{c^2+b+a}}$

Giải:

Áp dụng BĐT Cauchy-Schwarz, ta có $\sqrt{(a^2+b+c)(1+b+c)}\geq a+b+c$

Suy ra $\sum \frac{a}{\sqrt{a^2+b+c}}\leq \sum \frac{a\sqrt{b+c+1}}{a+b+c}$

Lại áp dụng BĐT Cauchy-Schwarz, ta có $(\sum a\sqrt{b+c+1})^2=(\sum \sqrt{a}.\sqrt{a(b+c+1)})^2\leq (a+b+c)(\sum a(b+c+1))=(a+b+c)[2(ab+bc+ca)+a+b+c]\leq (a+b+c)(a+b+c+\frac{2}{3}(a+b+c)^2)$

Suy ra $P\leq \sqrt{1+\frac{2}{3}(a+b+c)}\leq \sqrt{1+2}=\sqrt{3}$

Cách khác:

Không mất tính tổng quát,giả sử $a\geq c\geq b$

Ta sẽ CM 

$\frac{b}{\sqrt{b^{2}+c+a}}+\frac{c}{\sqrt{c^{2}+a+b}}\geq \frac{b+c}{a^{2}+b+c+a\sqrt{a^{2}+b+c}}$ (*******)

Ta có : $\frac{b}{\sqrt{b^{2}+c+a}}+\frac{c}{\sqrt{c^{2}+a+b}}\geq \frac{b}{\sqrt{c^{2}+c+a}}+\frac{c}{\sqrt{c^{2}+c+a}} =\frac{b+c}{\sqrt{c^{2}+c+a}}$

Và $a^{2}+b+c+a\sqrt{a^{2}+b+c}\geq a^{2}+b+c+a$ (do a không lớn hơn 1 theo giả sử) suy ra $1\leq \sqrt{c^{2}+c+a}\leq c^{2}+c+a+b$

Vậy là đã chứng minh được (*********)-dấu đẳng thức xảy ra tương đương b=c ,lại thấy vế phải của (*********) là 

$\frac{b+c}{a^{2}+b+c+a\sqrt{a^{2}+b+c}}= 1-\frac{a}{\sqrt{a^{2}+b+c}}$ nên 

$\sum \frac{a}{\sqrt{a^{2}+b+c}}\geq 1$

Đẳng thức xảy ra khi và chỉ khi trong (a,b,c) có 2 số $=0,1$ số $= \sqrt{3}$

 

(Hết trang 7)



#6
CD13

CD13

    Thượng úy

  • Thành viên
  • 1456 Bài viết

Bài 61:

Chứng minh: $(1+\frac{4a}{b+c})$$(1+\frac{4b}{a+c})$$(1+\frac{4c}{a+b})$ $>$25

Biết $a,b,c>0$

Giải:

Vì vai trò của $a,b,c$ là như nhau cho nên ta có thể giả sử rằng: $0<a\leq b\leq c$

 

Đặt $S=a+b+c$, bất đẳng thức cần chứng minh tương đuơng với:

 

$(S+3a)(S+3b)(S+3c)>25S(S-a)(S-b)(S-c)$

 

$\Leftrightarrow S^{3}+2S^{2}(a+b+c)+9S(ab+bc+ca)+27abc>25S^{3}-25S^{2}(a+b+c)+25S(ab+bc+ca)-25abc$

 

$\Leftrightarrow 4S^{3}-16S(ab+bc+ca)+52abc>0$

 

$\Leftrightarrow S(S^{2}-4(ab+bc+ca))+13abc>0$

 

$\Leftrightarrow S((a+b-c)^{2}-4ab)+13abc>0$

 

$\Leftrightarrow S(a+b-c)^{2}+ab(13c-4S)>0$

 

Bất đẳng thức cuối cùng đúng vì $13c-4S=13c-4(a+b+c)=9c-4(a+b)>0$

 

 

 

Bài 62:

Cho $a,b,c >0$ $\sum \frac{(2a+b+c)^2}{2a^2+(b+c)^2}\leq 8$

Giải:

Do bất đẳng thức đã cho là thuần nhất nên ta có thể chuẩn hóa $a+b+c=3$

Do đó bất đẳng thức trở thành 

                     $\sum \frac{(a+3)^2}{2a^2+(3-a)^2} \leq 8\Leftrightarrow \sum \frac{a^2+6a+9}{a^2-2a+3} \leq 24$

Đến đây ta dùng phương pháp hệ số bất định như sau : 

Giả sử $\frac{a^2+6a+9}{a^2-2a+3} \leq k(a-1)+8$

 $\Leftrightarrow ka^3+(7-3k)a^2+(5k-22)a+15-3k \geq 0$

Ta phải viết $ka^3+(7-3k)a^2+(5k-22)a+15-3k=(a-1)^2(ka+15-3k)$

Khai triển, đồng nhất hệ số ta được $k=4$

Vậy ta có $\frac{a^2+6a+9}{a^2-2a+3} \leq 4(a-1)+8$ với mọi $a>0$

Tương tự ta cũng có $\frac{b^2+6b+9}{b^2-2b+3} \leq 4(b-1)+8$

                                  $\frac{c^2+6c+9}{c^2-2c+3} \leq 4(c-1)+8$

Cộng 3 bất đẳng thức trên lại và để ý $a+b+c=3$, ta có được

                 $\sum \frac{a^2+6a+9}{a^2-2a+3} \leq 24$

Đẳng thức xảy ra khi $a=b=c>0$

 

 

 

Bài 63:

Cho ba số thực dương a,b,c. CMR $\sum \frac{a\sqrt{b}}{\sqrt{a}+2\sqrt{b}}\leq \frac{a+b+c}{3}$

 

Giải:

$\frac{a\sqrt{b}}{\sqrt{a}+2\sqrt{b}}\leq \frac{5a+b}{18}\Leftrightarrow \frac{(5\sqrt{a}+2\sqrt{b})(\sqrt{a}-\sqrt{b})^2}{18(\sqrt{a}+2\sqrt{b})}\geq 0$ ( đúng vì $a,b>0$ )

Tương tự $....$

$\Rightarrow \sum \frac{a\sqrt{b}}{\sqrt{a}+2\sqrt{b}} \leq \frac{6(a+b+c)}{18}=\frac{a+b+c}{3}$

Dấu $"="$$\Leftrightarrow a=b=c$

Cách khác:

Ta có:

$\sum \dfrac{a\sqrt{b}}{\sqrt{a}+2\sqrt{b}} \le \dfrac{a+b+c}{3}$

$\Longleftrightarrow \dfrac{2a\sqrt{b}}{\sqrt{a}+2\sqrt{b}}\le \dfrac{2(a+b+c)}{3}$

$\Longleftrightarrow \left (a-\dfrac{a\sqrt{a}}{2\sqrt{b}+\sqrt{a}} \right ) \le \dfrac{2(a+b+c)}{3}$

$\Longleftrightarrow \dfrac{a\sqrt{a}}{2\sqrt{b}+\sqrt{a}} \ge \dfrac{a+b+c}{3}$

Áp dụng bất đẳng thức Cauchy-Schwartz,ta có

$\sum \left (\dfrac{a\sqrt{a}}{2\sqrt{b}+\sqrt{a}} \right ) = \sum \left (\dfrac{a^2}{2\sqrt{ab}+a} \right ) \ge \dfrac{(a+b+c)^2}{(\sqrt{a}+\sqrt{b}+\sqrt{c})^2} \ge \dfrac{a+b+c}{3}$

Cách khác:

Sử dụng bất đẳng thức $\frac{9}{x+y+z}\leqslant \frac{1}{x}+\frac{1}{y}+\frac{1}{z}$ ta có

            $\frac{9}{\sqrt{a}+2\sqrt{b}}\leqslant \frac{1}{\sqrt{a}}+\frac{2}{\sqrt{b}}$

$\Rightarrow \frac{9a\sqrt{b}}{\sqrt{a}+2\sqrt{b}}\leqslant \frac{a\sqrt{b}}{\sqrt{a}}+\frac{2a\sqrt{b}}{\sqrt{b}}=\sqrt{ab}+2a$

Tương tự 2 bất đẳng thức còn lại rồi cộng vào ta được

           $\sum \frac{9a\sqrt{b}}{\sqrt{a}+2\sqrt{b}}\leqslant \sqrt{ab}+\sqrt{bc}+\sqrt{ac}+2(a+b+c)\leqslant 3(a+b+c)$

 $\sum \frac{a\sqrt{b}}{\sqrt{a}+2\sqrt{b}}\leqslant \frac{a+b+c}{3}$

Đẳng thức xảy ra khi $a=b=c>0$

 

 

 

Bài 64:

Cho $\left\{\begin{matrix} a,b,c\geq 0 & \\a+b+c=1 & \end{matrix}\right.$

 

CMR
$\sum \sqrt{a^{2}+2ab+2b^{2}}\geq \sqrt{5}$

Giải:

Đpcm$\Leftrightarrow \sum \sqrt{(a+b)^{2}+b^{2}}\geq \sqrt{5}$

Ta có $(a+b)^{2}+b^{2}=\frac{1}{5}[(a+b)^{2}+b^{2}](1^{2}+2^{2})\geq \frac{1}{5}[2(a+b)+b]^{2}=\frac{1}{5}(2a+3b)^{2}$

$\Rightarrow \sqrt{(a+b)^{2}+b^{2}}\geq \frac{1}{\sqrt{5}}(2a+3b)$

Hoàn toàn tương tự: $\sqrt{(b+c)^{2}+c^{2}}\geq \frac{1}{\sqrt{5}}(2b+3c)$

                                 $\sqrt{(c+a)^{2}+a^{2}}\geq \frac{1}{\sqrt{5}}(2c+3a)$

Cộng vế với vế của các BĐT cùng chiều ta có đpcm

Dấu bằng đạt $\Leftrightarrow a=b=c=\frac{1}{3}$

Cách khác:

$\sqrt{a^2+2ab+2b^2}\geq \frac{2a+3b}{\sqrt{5}}\Leftrightarrow (a-b)^2\geq 0$ ( luôn đúng $\forall a,b$ )

Tương tự ...

Suy ra $\sum \sqrt{a^{2}+2ab+2b^{2}}\geq \frac{5(a+b+c)}{\sqrt{5}}=\frac{5}{\sqrt{5}}=\sqrt{5}$

Dấu bằng xảy ra khi $a=b=c=\frac{1}{3}$

 

 

 

Bài 65:

Cho các số không âm a,b,c thỏa mãn $a+b+c=3$.Chứng minh rằng

$$\frac{a}{b^3+16}+\frac{b}{c^3+16}+\frac{c}{a^3+16}\geq\frac{1}{6}$$

Giải:

Ta có : $\sum \dfrac{a}{b^3+16}$=$\sum \dfrac{a}{16}-\sum \dfrac{ab^3}{16(b^3+16)}$ $\geq \dfrac{a+b+c}{16}-\dfrac{ab^2+bc^2+ca^2}{192}$

Vậy ta chỉ cần chứng minh:

$$ab^2+bc^2+ca^2 \leq 4$$

Đây là 1 bổ đề quen thuộc:

$$ab^2+bc^2+ca^2+abc\leq \dfrac{4(a+b+c)^3}{27}$$

Chứng minh: giả sử $b$ là số nằm giữa $a$ và $c$.

Suy ra: $a(a-b)(b-c)\geq 0$ $\Leftrightarrow abc+a^2b \geq ab^2+ca^2$

Ta có: $ab^2+bc^2+ca^2+abc \leq bc^2+2abc+a^2b=b(a+c)^2 \leq \dfrac{4(a+b+c)^3}{27}$

Bất đẳng thức được chứng minh.

Dấu đẳng thức xảy ra khi $(a,b,c)=(0,1,2)$ cùng các hoán vị.

 

 

 

Bài 66:

Cho a,b,c là số dương.Tìm min của P= $\frac{a+b}{a+b+c}+\frac{b+c}{4a+b+c}+\frac{c+a}{16b+c+a}$

Giải:

Đặt: $\left\{\begin{matrix} a+b+c=x & & \\ 4a+b+c=2y & & \\ 16b+c+a=6z& & \\ \end{matrix}\right.\Rightarrow \left\{\begin{matrix} a=\frac{2y-x}{3} & & \\ b=\frac{6z-x}{15} & & \\ c=\frac{21x-10y-6z}{15} & &\\ \end{matrix}\right.$

$\Rightarrow \left\{\begin{matrix} a+b=\frac{10y+6z-6x}{15} & & \\ b+c=\frac{4x-2y}{3} & & \\ c+a=\frac{16x-6z}{15} \end{matrix}\right.$

Khi đó ta có: $\left\{\begin{matrix} \frac{a+b}{a+b+c}=\frac{10y+6z-6x}{15x}=\frac{2y}{3x}+\frac{2z}{5x}-\frac{2}{5} & & \\ \frac{b+c}{4a+b+c}=\frac{4x-2y}{6y}=\frac{2x}{3y}-\frac{1}{3} & & \\ \frac{c+a}{16b+c+a}=\frac{16x-6z}{90z}=\frac{8x}{45z}-\frac{1}{15} & &\\ \end{matrix}\right.$

Từ đó, ta viết lại $P$ thành:

$P=\left ( \frac{2x}{3y}+\frac{2y}{3x} \right )+\left ( \frac{2z}{5x}+\frac{8x}{45z} \right )-\frac{4}{5}\geq \frac{4}{3}+\frac{8}{15}-\frac{4}{5}=\frac{16}{15}$

Vậy GTNN của $P$ là $\frac{16}{15}$

Đẳng thức xảy ra là bao gồm tất cả bộ số $(x, y, z)$ thỏa mãn hệ phương trình: $\left\{\begin{matrix} \frac{x}{y}=\frac{y}{x} & & \\ \frac{2z}{5x}=\frac{8x}{45z} & & \end{matrix}\right.$

 

 

Bài 67:

Cho 2 số thực khác nhau $a;b$. Chứng minh

 

$$\frac{(a-b)^4}{a^4+6a^2b^2+b^4}+\frac{4ab}{(a-b)^2} \geq 1$$

Giải:

Sử dụng hằng đẳng thức ta có 

BĐT $\Leftrightarrow \frac{a^4+b^4+6a^2b^2-4ab(a^2+b^2)}{a^4+6a^2b^2+b^4}+\frac{4ab}{(a-b)^2} \geqslant 1$

         $\Leftrightarrow \frac{ab}{(a-b)^2} \geqslant \frac{ab(a^2+b^2)}{a^4+b^4+6a^2b^2}$

         $\Leftrightarrow ab(a^4+b^4+6a^2b^2) \geqslant ab(a^2+b^2)(a-b)^2$

         $\Leftrightarrow ab\left [ a^4+b^4+6a^2b^2-(a-b)^2(a^2+b^2) \right ] \geqslant 0$

         $\Leftrightarrow 2a^2b^2.(2ab+a^2+b^2) \geqslant 0$

         $\Leftrightarrow 2a^2b^2(a+b)^2 \geqslant 0$

Bất đẳng thức trên luôn đúng với mọi $a,b$

Đẳng thức xảy ra khi $a=0,b=0$ hoặc $a+b=0$

 

 

 

Bài 68:

Cho a , b , c là ba số thực dương và a + b + c = 1 . Tìm GTLN của :
\[P = \sqrt {\frac{{ab}}{{\left( {1 - a} \right)\left( {1 - b} \right)}}}  + \sqrt {\frac{{bc}}{{\left( {1 - b} \right)\left( {1 - c} \right)}}}  + \sqrt {\frac{{ca}}{{\left( {1 - c} \right)\left( {1 - a} \right)}}} \]

Giải:

Áp dụng bất đẳng thức Cauchy ta có:
$P\leq \frac{1}{2}(\frac{a}{1-b}+\frac{b}{1-a}+\frac{b}{1-c}+\frac{c}{1-b}+\frac{c}{1-a}+\frac{a}{1-c})=\frac{1}{2}(\frac{1-a}{1-a}+\frac{1-b}{1-b}+\frac{1-c}{1-c})=\frac{3}{2}$.

Dấu bằng xảy ra khi a=b=c=1/3.

 

 

 

Bài 69:

Ch0 $a,b,c$ thỏa mãn $a,b,c \in \left [ 1;2 \right ]$

Chứng minh rằng $a^3+b^3+c^3 \leqslant 5abc$

Giải:

Viết lại bất đẳng thức cần chứng minh thành: $P=\frac{a^{2}}{bc}+\frac{b^{2}}{ca}+\frac{c^{2}}{ab}\leq 5$

 

Do vai trò bình đẳng của $a,b,c$ nên giả sử: $1\leq a\leq b\leq c\leq 2$. Khi đó: $(a-b)(b^{2}-c^{2})\geq 0$

 

                                     $\Leftrightarrow b^{3}\leq ab^{2}+bc^{2}-ac^{2}$

 

                                     $\Leftrightarrow \frac{b^{2}}{ca}\leq \frac{b}{c}+\frac{c}{a}-\frac{c}{b}$           

 

Mặt khác, ta có: $\frac{a^{2}}{bc}\leq \frac{a^{2}}{ac}=\frac{a}{c}$ và $\frac{c^{2}}{ab}\leq \frac{2c}{ab}=\frac{2c}{b}$

 

Cộng vế với vế ba bất đẳng thức trên ta có: $P\leq \left ( \frac{b}{c}+\frac{c}{b} \right )+\left ( \frac{a}{c}+\frac{c}{a} \right )$

 

Vì $b\leq c\leq 2\leq 2b$ nên $\frac{2b}{c}\geq 1, \frac{c}{b}\geq 1>\frac{1}{2}$

 

Do đó: $\left ( \frac{2b}{c}-1 \right )\left ( \frac{c}{b}-\frac{1}{2} \right )\geq 0\Rightarrow \frac{b}{c}+\frac{c}{b}\leq \frac{5}{2}$

 

Tương tự ta có: $\frac{a}{c}+\frac{c}{a}\leq \frac{5}{2}$

 

Do đó ta có: $P\leq 5$

 

Đẳng thức xảy ra khi và chỉ khi $a=b=1, c=2$ và các hoán vị

Cách khác:

Giả sử $a\geq b\geq c$

Dễ thấy $(a-2))(a^{2}+2a-1)\leq 0\Leftrightarrow a^{3}+2\leq 5a$

$(b-1)(b^{2}+b+1-5a)\leq 0\Leftrightarrow 5a+b^{3}\leq 5ab+1$

$(c-1)(c^{2}+c+1-5ab)\leq 0\Leftrightarrow 5ab+c^{3}\leq 5abc+1$

 (Vì $b^{2}+b+1\leq a^{2}+a+1\leq 5a$

 

 

 

Bài 70:

Cho a,b,c là các số dương và $ab+bc+ca=2abc$ Chứng minh rằng:

$\frac{1}{a(2a-1)^{2}}+\frac{1}{b(2b-1)^{2}}+\frac{1}{c(2c-1)^{2}}\geq \frac{1}{2}$

Giải:

ĐK : $a,b,c\neq \frac{1}{2}$ . Chia cả 2 vế của giả thiết cho $ abc \neq 0 $ ta được $\frac{1}{a}+\frac{1}{b}+\frac{1}{c}=2$

$\frac{1}{a(2a-1)^2}\geq \frac{1}{a}-\frac{1}{2}$

$\Leftrightarrow \frac{2+(a-2)(2a-1)^2}{2a(2a-1)^2}\geq 0$ $\Leftrightarrow \frac{a(2a-3)^2}{2a(2a-1)^2}\geq 0$ Điều này đúng vì $a\neq \frac{1}{2}$ ; $a> 0$

Dấu bằng xảy ra khi $a=b=c=\frac{3}{2}$

 

 

 

Bài 71:

Cho $a,b,c$ dương và $a+b+c=2013$ Tìm GTLN của $T=\frac{(2013-a)(2013-b)(2013-c)}{(2013+a)(2013+b)(2013+c)}$

Giải:

$T=\frac{(a+b)(b+c)(c+a)}{(2a+b+c)(2b+c+a)(2c+a+b)}$

Đặt $x=a+b,y=b+c,z=c+a$

Ta có $T=\frac{xyz}{(x+y)(y+z)(z+x)}\leq \frac{1}{8}$

Dấu "$=$" xảy ra $\Leftrightarrow a=b=c=671$

 

 

Bài 72:

Cho $a,b,c\geq 0$ thoả mãn $a^{2}+b^{2}+c^{2}\leq 3b$.

Tìm GTNN: P= $\frac{1}{(a+1)^{2}}+\frac{4}{(b+2)^{2}}+\frac{8}{(c+3)^{2}}$

Giải:

Áp dụng BĐT $\frac{1}{t^2}+\frac{1}{p^2}\geq \frac{8}{(t+p)^2}$, ta có $P=\frac{1}{(a+1)^2}+\frac{1}{(\frac{b}{2}+1)^2}+\frac{8}{(c+3)^2}\geq \frac{8}{(a+\frac{b}{2}+2)^2}+\frac{8}{(c+3)^2}\geq \frac{64}{(a+\frac{b}{2}+c+5)^2}$

(1)

Từ giả thiết, ta có $3b+6\geq (a^2+1)+(b^2+4)+(c^2+1)\geq 2a+4b+2c\Rightarrow 6\geq 2a+b+2c\Rightarrow 3\geq a+\frac{b}{2}+c$             

(2)

Từ (1) và (2) suy ra GTNN của P là 8

 

 

 

Bài 73:

Cho x,y,z là các số thực dương thỏa mãn xyz=1.chứng minh rằng :

$\frac{x+y}{x+y+1}+\frac{y+z}{y+z+1}+\frac{z+x}{z+x+1}\geq 2$

Giải:

Ta có: $\frac{x+y}{x+y+1}=1-\frac{1}{x+y+1}$

 

Do đó bất đẳng thức cần chứng minh tương đương với: 

 

$\frac{1}{x+y+1}+\frac{1}{y+z+1}+\frac{1}{z+x+1}\leq 1$

 

Đặt $x=a^{3},y=b^{3},z=c^{3}$, ta có $abc=1$ và bất đẳng thức trên tương đương:

 

$\frac{1}{1+a^{3}+b^{3}}+\frac{1}{1+b^{3}+c^{3}}+\frac{1}{1+c^{3}+a^{3}}\leq 1$

 

Ta có: $a^{3}+b^{3}=(a+b)(a^{2}-ab+b^{2})\geq ab(a+b)$

 

Do đó: $a^{3}+b^{3}+1\geq ab(a+b)+abc=ab(a+b+c)$ nên $\frac{1}{1+a^{3}+b^{3}}\leq \frac{c}{a+b+c}$

 

Thiết lập hai bất đẳng thức tương tự rồi cộng vế với vế ta có đpcm

 

 

Bài 74:

Cho a,b,c,d là các số thực dương và abcd=1. Tìm Max P=$\frac{1}{3+a}+\frac{1}{3+b}+\frac{1}{3+c}+\frac{1}{3+d}$

Giải:

Ta xét hàm $f(x)=\frac{1}{3+x}+\dfrac{1}{16} \ln x-\frac{1}{4}$
$f'(x)=0$ khi và chỉ khi $x=1$ hoặc $x=9$
Nếu $0<x < 9$ thì $f(x) \leq f(1)=0$
Suy ra $\frac{1}{3+x}+\dfrac{1}{16} \ln x-\frac{1}{4} \leq 0$
_________________
Tóm lại: Nếu $a,b,c,d$ có 1 số $>9$ (giả sử là $d>9$) thì $P \leq \frac{3}{4}-\dfrac{1}{16} \ln \frac{1}{d}+\frac{1}{3+d}<1$
Nếu $a,b,c,d$ có 2 số $>9$ (giả sử là $c,d>9$) thì $P \leq \frac{1}{2}-\dfrac{1}{16} \ln \frac{1}{cd}+\frac{1}{3+c}+\frac{1}{3+d}<1$
Nếu $a,b,c,d$ có 3 số $>9$ (giả sử là $b,c,d>9$) thì luôn có $P<1$
Nếu $a,b,c,d$ có 4 số $>9$ thì luôn cũng có $P<1$
Nếu $a,b,c,d$ đều $<9$ thì theo bổ đề trên $P \leq 1-\frac{1}{16} \ln 1=1$
Suy ra OK

 

 

 

Bài 75:

Cho $a, b, c >0$ và $a+b+c=3$ tìm max

 

$\frac{2}{3+ab+bc+ca}+\sqrt[3]{\frac{abc}{(1+a)(1+b)(1+c)}}$

 

Giải:

Từ giả thiết suy ra:

$$abc\leq \left ( \frac{a+b+c}{3} \right )^3=1$$

Lại có:

$$\left ( ab+bc+ca \right )^2\geq 3abc\left ( a+b+c \right )=9abc$$

$$\Rightarrow ab+bc+ca\geq 3\sqrt{abc}$$

Ta có:

$$P=\frac{2}{3+ab+bc+ca}+\sqrt[3]{\frac{abc}{\left ( 1+a \right )\left ( 1+b \right )\left ( 1+c \right )}}$$

$$=\frac{2}{3+ab+bc+ca}+\sqrt[3]{\frac{abc}{4+ab+bc+ca+abc}}$$

$$\leq \frac{2}{3+3\sqrt{abc}}+\sqrt[3]{\frac{abc}{4+3\sqrt{abc}+abc}}$$

Đặt:

$$f\left ( x \right )=\frac{2}{3+3\sqrt{x}}+\sqrt[3]{\frac{x}{4+3\sqrt{x}+x}},(x=abc,0<x \le 1)$$

Khi đó, ta có:

$f'(x)>0$

$\Rightarrow$ Hàm $f$ đồng biến trong $(0;1]$

Do đó:

$max P=max f(x)=\frac{5}{6}$

Đẳng thưc xảy ra $\Leftrightarrow x=1 \Leftrightarrow a=b=c=1$

Cách khác:

$\frac{2}{3+ab+bc+ca}+\sqrt[3]{\frac{abc}{(1+a)(1+b)(1+c)}}$
$=\frac{2}{9}.\frac{(a+b+c)^2}{(a+1)b+(b+1)c+(c+1)a}+\sqrt[3]{\frac{abc}{(1+a)(1+b)(1+c)}}$

$\leq\frac29.\sum\frac{1}{1+a}+\frac39\sum\frac{a}{1+a}=\frac23+\frac19.\sum\frac{a}{1+a}$

$\leq\frac23+\frac{1}{18}.\sum\sqrt{a}\leq\frac23+\frac{1}{18}.\sqrt{3(a+b+c)}=\frac56$

 

 

 

Bài 76:

Cho $a,b,c \geq 0$ thoả mãn $a^2+b^2+c^2=3$

Tìm Min của $P=\frac{(a+b+c-1)^2}{a^2b+b^2c+c^2a}+\frac{1}{a}+\frac{1}{b}+\frac{1}{c}$

Giải:

Ta có $(a^2+b^2+c^2)(a+b+c)\geq 3(a^2b+b^2c+c^2a)$

$\Rightarrow a+b+c\geq a^2b+b^2c+c^2a$ (vì $a^2+b^2+c^2=3$)

$P\geq \frac{(a+b+c-1)^2}{a+b+c}+\frac{9}{a+b+c}=\sum a+\frac{9}{\sum a}+\frac{1}{\sum a}-2$

Ta lại có $\sum a\leq \sqrt{3\sum a^2}=3$

Suy ra $P_{Min}=\frac{13}{3}$ dấu $"="$ $\Leftrightarrow a=b=c=1$

 

 

 

Bài 77:

Cho $ a,b,c>0$ thỏa mãn $a+b+c=1$. Chứng minh rằng:

$$\frac{a^2}{9a+1}+\frac{b^2}{9b+1}+\frac{c^2}{9c+1} \leq \frac{1}{12\sqrt{3(ab+bc+ca)}}$$

Giải:

Ta có: $\frac{a^{2}}{9a+1}=\frac{a}{9}-\frac{a}{9(9a+1)}$ nên bất đẳng thức cần chứng minh tương đương với:

 

    $\frac{a}{9a+1}+\frac{b}{9b+1}+\frac{c}{9c+1}+\frac{3}{4\sqrt{3(ab+bc+ca)}}\geq 1$

 

Áp dụng bất đẳng thức Cauchy - Schwarz và AM-GM ta có:

 

   $\frac{a}{9a+1}+\frac{b}{9b+1}+\frac{c}{9c+1}\geq \frac{(a+b+c)^{2}}{a(9a+1)+b(9b+1)+c(9c+1)}=\frac{1}{9(a^{2}+b^{2}+c^{2})+1}$

 

  $\frac{3}{4\sqrt{3(ab+bc+ca)}}\geq \frac{3}{6(ab+bc+ca)+2}$

 

Áp dụng bất đẳng thức Cauchy - Schwarz ta có:

 

  $\frac{1}{9(a^{2}+b^{2}+c^{2})+1}+\frac{3}{6(ab+bc+ca)+2}\geq \frac{(1+3)^{2}}{9(a+b+c)^{2}+7}=1$

 

Do đó ta có điều phải chứng minh.

 

 

Bài 78:

Cho $x,y,z$ thực không âm thỏa $z=\max \{x,y,z\}$ và $xy+xz+yz>0$. Tìm min của biêu thức $$P=\frac{x}{y+z}+2\sqrt{\frac{y}{x+z}}+3\sqrt[3]{\frac{z}{x+y}}$$

 

Giải:

 Để ý với điều kiện $x,y,z$ không âm, ta nghĩ ngay đến chuyện dấu bằng xảy ra khi có 1 số bằng 0. Lần mò thêm tý nữa, ta có thể dự đoán dấu bằng xảy ra khi $y=0,z=x$ và $P=4$.

 

Lại nhìn vào biểu thức $P$ và điều kiện $z=max\{x;y;z\}$, thật tự nhiên ta nghĩ đến việc đưa biểu thức về 1 biến để khảo sát hàm số, cụ thể là $\sqrt[3]{\frac{z}{x+y}}$. Số hạng này mang tính đối xứng giữa 2 biến $x,y$ nên ta cũng sẽ đi tìm cách đánh giá đưa $P$ về đối xứng the0 $x,y$ bằng việc sử dụng 1 số bất đẳng thức quen thuộc :)

Áp dụng liên tiếp AM-GM ta có :

$$\frac{x}{y+z}+2\sqrt{\frac{y}{x+z}}\geq 2\left(\sqrt{\frac{x}{y+z}}+\sqrt{\frac{y}{x+z}}\right)-1$$

$$=4\left(\frac{x}{2\sqrt{x(y+z)}}+\frac{y}{2\sqrt{y(x+z)}}\right)-1$$

$$\geq \frac{4(x+y)}{x+y+z}-1=\dfrac{4}{1+\frac{z}{x+y}}-1$$

Giờ thì đặt $\frac{z}{x+y}=t^3$ với $t\geq \sqrt[3]{\frac{1}{2}}$ ta có:

$$P\geq \frac{4}{1+t^3}+3t-1=f(t)$$

Ta có $f'(t)=4.\frac{(t-1)(t^2+t-1)(t^3+2t+1)}{(t^3+1)^2}$  nhưng do $t^2+t-1>0$ (Do $t\geq \sqrt[3]{\frac{1}{2}}$) nên $f(t)$ đạt min khi $t=1$ và $P=4$

Kết thúc chứng minh, đẳng thức xảy ra tại $x=z,y=0$ $\blacksquare$

 

 

 

Bài 79:

Tìm minP theo 2 cách:

 

$P=\frac{1}{2+4a}+\frac{1}{3+9b}+\frac{1}{6+36c}$

 

trong đó a,b,c là 3 số thực dương thoả mãn $a+b+c=1$

Giải:

Áp dụng bất đẳng thức $Cauchy,$ ta có:

$$\frac{1}{2+4a}+\frac{2+4a}{16}\geq \frac{1}{2}$$

$$\frac{1}{3+9a}+\frac{3+9a}{36}\geq \frac{1}{3}$$

$$\frac{1}{6+36a}+\frac{6+36a}{144}\geq \frac{1}{6}$$

Do đó: $$\frac{1}{2+4a}+\frac{1}{3+9b}+\frac{1}{6+36c}+\frac{2+4a}{16}+\frac{3+9a}{36}+\frac{6+36a}{144}\geq 1$$

$$\Leftrightarrow P=\frac{1}{2+4a}+\frac{1}{3+9b}+\frac{1}{6+36c}\geq 1-\left (\frac{2+4a}{16}+\frac{3+9a}{36}+\frac{6+36a}{144} \right )$$

$$\Leftrightarrow P=\frac{1}{2+4a}+\frac{1}{3+9b}+\frac{1}{6+36c}\geq 1-\frac{1}{4}-\frac{a+b+c}{4}=\frac{1}{2}$$

Vậy giá trị nhỏ nhất của $P$ là $\frac{1}{2}$ khi $a=\frac{1}{2};\ b=\frac{1}{3};\ c=\frac{1}{6}$

 

 

 

Bài 80:

Cho các số dương $a,b,c$ thoả mãn $ab^2+bc^2+ca^2=3$ .Chứng minh rằng:

$\sum \frac{2a^5+3b^5}{ab} \geq 15(a^3+b^3+c^3-2)$

Giải:

Bài này chỉ phức tạp ở công thức biến đổi: Bất đẳng thức cần chứng minh tương đương:

$\sum \frac{(a-b)^{4}(2a+3b)}{ab}\geq 0$ và tất nhiên là đúng (Bạn có thể tìm cách biến đổi hoặc nếu chưa chắc thì nhân tung biểu thức đó ra sẽ ra biểu thức ban đầu, mình thử rồi)

Trang 18

 

 

 

 

 



#7
dera coppy

dera coppy

    Binh nhất

  • Thành viên
  • 44 Bài viết

Bài 81:

Cho a,b,c,d thỏa mãn $0 \leq a,b,c,d \leq 1$

Tìm Max $M=\frac{a}{bcd+1}+\frac{b}{cda+1}+\frac{c}{abd+1}+\frac{d}{abc+1}$



#8
hoctrocuaHolmes

hoctrocuaHolmes

    Thượng úy

  • Thành viên
  • 1013 Bài viết

Bài 81:

Cho a,b,c,d thỏa mãn $0 \leq a,b,c,d \leq 1$

Tìm Max $M=\frac{a}{bcd+1}+\frac{b}{cda+1}+\frac{c}{abd+1}+\frac{d}{abc+1}$

Ta có 

$\frac{a}{bcd+1}+\frac{b}{cda+1}+\frac{c}{abd+1}+\frac{d}{abc+1}=\sum \frac{a^{2}}{abcd+a}\geq \frac{(a+b+c+d)^{2}}{4abcd+(a+b+c+d)}$

Lại có $4abcd\leq \frac{(a+b+c+d)^{4}}{64}\Leftrightarrow 4abcd+(a+b+c+d)\leq \frac{(a+b+c+d)^{4}+64(a+b+c+d)}{64}$

$\Leftrightarrow \frac{(a+b+c+d)^{2}}{4abcd+(a+b+c+d)}\geq \frac{64(a+b+c+d)^{2}}{(a+b+c+d)[((a+b+c+d)^{3}+64)]})=\frac{64(a+b+c+d)}{(a+b+c+d)^{3}+64}$

$=\frac{64(1+1+1+1)}{(1+1+1+1)^{3}+64}=2

Dấu ''='' xảy ra $\Leftrightarrow a=b=c=d=1$


Bài viết đã được chỉnh sửa nội dung bởi votruc: 16-05-2015 - 17:01


#9
an1907

an1907

    Trung sĩ

  • Thành viên
  • 181 Bài viết

$Cho a , b , c thoả mãn : a + b + c = 3 . Chứng minh : A = \frac{a^{2}}{a + 2b^{2}} + \frac{b^{2}}{b + 2c^{2}} + \frac{c^{2}}{c + 2a^{2}} \geq 1$



#10
quan1234

quan1234

    Thượng sĩ

  • Thành viên
  • 257 Bài viết

$Cho a , b , c thoả mãn : a + b + c = 3 . Chứng minh : A = \frac{a^{2}}{a + 2b^{2}} + \frac{b^{2}}{b + 2c^{2}} + \frac{c^{2}}{c + 2a^{2}} \geq 1$

$\frac{a^2}{a+2b^2}+\frac{b^2}{b+2c^2}+\frac{c^2}{c+2a^2}= a+b+c-(\frac{2ab^2}{a+2b^2}+\frac{2bc^2}{b+2c^2}+\frac{2ca^2}{c+2a^2})\geq 3-(\frac{2(ab)^\frac{2}{3}}{3}+\frac{2(bc)^\frac{2}{3}}{3}+\frac{2(ac)^\frac{2}{3}}{3})$

Dễ CM được $(ab)^\frac{2}{3}+(bc)^\frac{2}{3}+(ac)^\frac{2}{3}\leq \frac{(a+ab+b)+(b+bc+c)+(a+ac+c)}{3}\leq 3$

Thay vào được đpcm


Bài viết đã được chỉnh sửa nội dung bởi quan1234: 28-08-2015 - 17:54


#11
an1907

an1907

    Trung sĩ

  • Thành viên
  • 181 Bài viết

Cho $a, b, c > 0$ thoả mãn $a + b + c = 2$ . Chứng minh rằng : $\frac{a}{2 + b - a} + \frac{b}{2 + c - b} + \frac{c}{2 + a - c} \geq 1$



#12
an1907

an1907

    Trung sĩ

  • Thành viên
  • 181 Bài viết

Với $0 \leq x, y, z \leq 1$ . Chứng minh rằng : $2\left ( x^{3} + y^{3} + z^{3} \right ) - \left ( x^{2}y + y^{2}z + z^{2}x \right ) \leq 3$



#13
an1907

an1907

    Trung sĩ

  • Thành viên
  • 181 Bài viết

Tìm GTLN và GTNN của biểu thức : $\left ( a + b + c \right )\left ( c + d + e \right )\left ( e + f + a \right )$ trong đó $a, b, c, d, e, f$ lấy các giá trị khác nhau trong tập hợp $\left \{ 1, 2, 3, 4, 5, 6 \right \}$



#14
an1907

an1907

    Trung sĩ

  • Thành viên
  • 181 Bài viết

Chứng minh rằng :

1. $\sqrt{a} + \sqrt[3]{a} + \sqrt[6]{a} \leq a + 2$ trong đó $a \geq 0$  

2. $\sqrt[4]{\left ( a + 1 \right )\left ( b + 4 \right )\left ( c - 2 \right )\left ( d - 3 \right )} \leq a + b + c + d$ với $a \geq -1 , b \geq -4 , c \geq 2 , d > 3$

3. $x^{2} + y^{2} + \frac{1}{x} + \frac{1}{y} \geq 2\left ( \sqrt{x} + \sqrt{y} \right )$ với $x, y > 0$

4. $\frac{a}{b + c + 1} + \frac{b}{a + c + 1} + \frac{c}{a + b + 1} + \left ( 1 - a \right )\left ( 1 - b \right )\left ( 1 - c \right ) \leq 1$ với $0 \leq a, b, c \leq 1$


Bài viết đã được chỉnh sửa nội dung bởi an1907: 28-08-2015 - 21:48


#15
quan1234

quan1234

    Thượng sĩ

  • Thành viên
  • 257 Bài viết

Với $0 \leq x, y, z \leq 1$ . Chứng minh rằng : $2\left ( x^{3} + y^{3} + z^{3} \right ) - \left ( x^{2}y + y^{2}z + z^{2}x \right ) \leq 3$

Vì $0\leq x,y,z\leq1$ nên $(x^2-1)(y-1)+(y^2-1)(z-1)+(z^2-1)(x-1)\geq0$$\Leftrightarrow -x^2y-y^2z-z^2x\leq 3-(x+y+z+x^2+y^2+z^2)\leq 3-2(x^3+y^3+z^3)\Rightarrow 3\geq 2(x^3+y^3+z^3)-x^2y-y^2z-z^2x$


Bài viết đã được chỉnh sửa nội dung bởi quan1234: 29-08-2015 - 13:11


#16
an1907

an1907

    Trung sĩ

  • Thành viên
  • 181 Bài viết

sao lại có bđt : $2\left ( x^{3} + y^{3} + z^{3} \right ) \leq x + y + z + x^{2} + y^{2} + z^{2}$ hả bạn??



#17
gianglqd

gianglqd

    Trung úy

  • Thành viên
  • 894 Bài viết

Mong ai đó tổng hợp lại thành file pdf cho dễ nhìn ạ


Mabel Pines - Gravity Falls

 

 

                                                        

 


#18
quan1234

quan1234

    Thượng sĩ

  • Thành viên
  • 257 Bài viết

sao lại có bđt : $2\left ( x^{3} + y^{3} + z^{3} \right ) \leq x + y + z + x^{2} + y^{2} + z^{2}$ hả bạn??

Vì $0\leq x,y,z\leq 1\Rightarrow x\geq x^3, x^2\geq x^3$ những cái còn lại tương tự



#19
an1907

an1907

    Trung sĩ

  • Thành viên
  • 181 Bài viết

Giả sử các số thực $a, b, c, d, x, y, z, t$ thoả mãn hệ : $a + b + c + d = 2, ax + by + cz + dt = 6$. Tìm GTNN của biểu thức :

$T = \sqrt{16a^{2} + a^{2}x^{2}} + \sqrt{16b^{2} + b^{2}y^{2}} + \sqrt{16c^{2} + c^{2}z^{2}} + \sqrt{16d^{2} + d^{2}t^{2}}$



#20
an1907

an1907

    Trung sĩ

  • Thành viên
  • 181 Bài viết

Chứng minh :

a, Nếu $a, b, c > 0$ sao cho $abc = 1$ thì : $\frac{1}{\left ( 1 + a \right )^{3}} + \frac{1}{\left ( 1 + b \right )^{3}} + \frac{1}{\left ( 1 + c \right )^{3}} + \frac{5}{\left ( 1 + a \right )\left ( 1 + b \right )\left ( 1 + c \right )} \geq 1$

b, Nếu $a, b, c \geq -1, a + b + c = 1$ thì : $\frac{a}{1 + a^{2}} + \frac{b}{1 + b^{2}} + \frac{c}{1 + c^{2}} \leq \frac{9}{10}$


Bài viết đã được chỉnh sửa nội dung bởi an1907: 30-08-2015 - 13:10





0 người đang xem chủ đề

0 thành viên, 0 khách, 0 thành viên ẩn danh